Главная Юзердоски Каталог Трекер NSFW Настройки

Математика

Ответить в тред Ответить в тред
Check this out!
<<
Назад | Вниз | Каталог | Обновить | Автообновление | 316 37 176
Копипасты тред Аноним 03/06/17 Суб 09:26:19 19669 1
maxresdefault.jpg 126Кб, 1280x960
1280x960
В этом итт треде мы будем спасать утопающие в гомотопическом хаосе золотые россыпи крупиц народной мудрости /math. Ответы на платиновые вопросы, рецензии на статьи Мочидзуки, исправленные доказательства из Зорича и просто любые хорошие, годные посты.

Правила:
1. Не копипастить свои посты.
2. Не постить мемасы и форсы — для этого есть деградации тред, тут собираем мудрость.
3. Мод, удаляй все, что не является копипастой, но не сразу, а через пару дней — так, чтобы обсуждение тонуло вниз, а копипасты всплывали наверх.
Аноним # OP 03/06/17 Суб 09:27:07 19670 2
Тредж, очевидно, тематический-модерируемый, если вдруг кто не понял.
Аноним 03/06/17 Суб 09:31:33 19671 3
В Хаскелле есть категории?

Эндофунктор - это когда категория отображается сама в себя.
А не-эндофунктор - это когда функтор устанавливается между двумя разными категориями.
В хаскелле есть только одна категория, Hask. Других категорий в нём не может быть в принципе. Поэтому все функторы в нём отображают Hask в Hask - переводят типы данных в другие типы данных, программы (стрелки) в программы.
Вот если бы Hask морфировала каким-то образом в теорию групп (типы данных переводились в группы, а программы - в гомоморфизмы групп), то это был бы не-эндофунктор.
Аноним 03/06/17 Суб 09:36:33 19672 4
Хочу не просто оперировать пределами, решая упражнения на них, а понимать их глубинный смысл.

Тогда только один путь: решай задачи на доказательства, пытайся по формулировкам теорем доказывать их сам, перед тем, как смотреть доказательства в учебнике, при прочтении определения пытайся сам придумать примеры определяемого объекта, выявить какие-то простейшие свойства. В идеале еще надо иметь кругозор и перед началом чтения книги примерно знать, какие результаты в этой книге самые главные, и когда встречаешь теорему/лемму/определение думать, как это может помочь для получения тех главных результатов.

В случае с пределами, например, тебе нужно поверхностно узнать про интеграл и производную (например, с точки зрения физической интуиции: разбили множество на бесконечно малые кусочки и просуммировали; скорость в определенный момент времени) и подумать, с помощью чего интегрирование и дифференцирование можно было бы строго обосновать. Тогда не будет возникать вопрос "зачем нужны пределы".

Если ты реально только-только начал заниматься математикой и читаешь первые главы Зорича, то, скорее всего, ты не сможешь делать все, что я перечислил, но к этому надо стремиться, если хочешь максимального понимания. Если у тебя в жизни нет времени, чтобы постоянно заниматься математикой, то тебе будет намного сложнее, придется либо забивать на осознанность вообще и как макака применять готовые формулы, либо научиться понимать, что концептуально, а что чисто техническим аспектом, но это приходит только с опытом и кругозором, так что первый год-два в любом случае придется изучать математику как чистые математики.
Аноним 03/06/17 Суб 17:30:41 19683 5
http://www.mccme.ru/edu/index.php?ikey=viarn_oprepmat
Математика --- часть физики. Физика --- экспериментальная, естественная наука, часть естествознания. Математика --- это та часть физики, в которой эксперименты дешевы.

Тождество Якоби (вынуждающее высоты треугольника пересекаться в одной точке) --- такой же экспериментальный факт, как то, что Земля кругла (т.е. гомеоморфна шару). Но обнаружить его можно с меньшими затратами.

В середине двадцатого века была предпринята попытка разделить математику и физику. Последствия оказались катастрофическими. Выросли целые поколения математиков, незнакомых с половиной своей науки и, естественно, не имеющих никакого представления ни о каких других науках. Они начали учить своей уродливой схоластической псевдоматематике сначала студентов, а потом и школьников (забыв о предупреждении Харди, что для уродливой математики нет постоянного места под Солнцем).

Поскольку ни для преподавания, ни для приложений в каких-либо других науках схоластическая, отрезанная от физики, математика не приспособлена, результатом оказалась всеобщая ненависть к математикам --- и со стороны несчастных школьников (некоторые из которых со временем стали министрами), и со стороны пользователей.

Уродливое здание, построенное замученными комплексом неполноценности математиками-недоучками, не сумевшими своевременно познакомиться с физикой, напоминает стройную аксиоматическую теорию нечетных чисел. Ясно, что такую теорию можно создать и заставить учеников восхищаться совершенством и внутренней непротиворечивостью возникающей структуры (в которой определена, например, сумма нечетного числа слагаемых и произведение любого числа сомножителей). Четные же числа с этой сектантской точки зрения можно либо объявить ересью, либо со временем ввести в теорию, пополнив ее (уступая потребностям физики и реального мира) некоторыми "идеальными" объектами.

К сожалению, именно подобное уродливое извращенное построение математики господствовало в преподавании математики в течение десятилетий. Возникнув первоначально во Франции, это извращение быстро распространилось на обучение основам математики сперва студентов, а потом и школьников всех специальностей (сперва во Франции, а потом и в других странах, включая Россию).

Ученик французской начальной школы на вопрос "сколько будет 2+3" ответил: "3+2, так как сложение коммутативно". Он не знал, чему равна эта сумма, и даже не понимал, о чем его спрашивают!

Другой французский школьник (на мой взгляд, вполне разумный) определил математику так: "там есть квадрат, но это нужно еще доказать".

По моему преподавательскому опыту во Франции, представление о математике у студентов (вплоть даже до обучающихся математике в Ecole Normale Superieure --- этих явно неглупых, но изуродованных ребят мне жалко больше всего) --- столь же убого, как у этого школьника.
...
Аноним 03/06/17 Суб 23:01:49 19694 6
Чем больше я думаю по этому поводу, тем больше мне кажется, что математика определяется основополагающими принципами и источниками человеческого разума, на стыке их производной в виде рассудочной деятельности и той части реальности, которая доступна для познания через органы чувств.

Математика - это заложенный самим Богом способ более глубокого, чем обыденное, познания окружающей действительности. Или случайно образованное продолжение обычного языка, расширенного для мышления отвлеченных, но естественным образом ставшими полезными/подвластными человеку свойств чувственно наблюдаемых объектов.

Во! Математика это короче духовность, а вы со своим онанизмом и бездушными вычислениями на мертвых машинах всё портите! DEUS VULT!
Аноним 03/06/17 Суб 23:05:30 19696 7
>>19694
Кстати, добавляйте заголовок жирным, чтобы пасты от непаст сразу можно было отличить и чтобы был хоть какой-то контекст.
Аноним 03/06/17 Суб 23:19:59 19698 8
Чем занимаются математики.
Бонус: бьющий мимо цели наезд на конструктивизм.
#основания #философия #смысл_жизни


Математика изучает воображаемые сущности. Эти сущности, согласно неокантианству, расположены в созерцательном пространстве, что даёт возможность делать о них априорные синтетические суждения. Эти суждения называются определениями. Применение к определениям аналитических рассуждений позволяет чётче осознать, чем же являются рассматриваемые (в созерцательном пространстве) объекты.

Проще говоря, математик сначала воображает что-то, затем делает несколько утверждений о воображенном объекте, затем из этих утверждений выводит следствия.

Поместить в созерцательное пространство и начать созерцать можно какую угодно идею. Но не всякие идеи обладают тем изящным свойством, что при их созерцании получается много аналитических суждений, сложным образом связанных друг с другом. Лишь те идеи, при воображении которых достоверно получается много нескучных теорем, изучаются математикой и называются математическими объектами.

Похожей деятельностью занимаются вообще все теоретики - все они воображают. Однако у математики есть всё-таки своя специфика. Говорить о ней я не буду, потому что это слишком интимная вещь. Продолжу разговор о созерцательном пространстве.

Обычно воображается не одна-единственная сущность, но целый сонм сущностей. Этот сонм называется универсумом. Сущности, входящие в универсум, однородны в следующем смысле. Каждое определение можно рассматривать как фильтр, сквозь который одни сущности универсума проходят, другие не проходят. Сущности, которые проходят через фильтр, неразличимы этим фильтром. Так вот, для всякого универсума есть определение, относительно которого все объекты универсума неразличимы. Для универсума теории групп таким определением является группа - все сущности универсума подходят под это определение. Для универсума теории колец таким определением является кольцо. Для метаматематики универсумом является сонм текстов. И т.п.

Для каких-то универсумов есть описания, позволяющие составить представление о сущностях, входящих в сонм, для каких-то универсумов таких описаний нет. Например, для универсума стандартной теории множеств есть описание: именно, под универсумом понимается кумулятивная иерархия фон Неймана. Этот универсум обозначается V. Вместе с тем, легко воображаются универсумы множеств, которые могут напоминать, а могут и не напоминать V. Эти универсумы описываются неклассическими теориями множеств.

Существование понимается относительно какого-то универсума. Если объект x существует относительно U, то это означает, что при воображении универсума U необходимо также иметь в виду и объект x. Несуществование x соответственно означает, что при воображении универсума не следует воображать x.

Конструктивизм занимается ровно этим же самым - исследует воображаемые сонмы сущностей. Однако делает это "со связанными руками", фанатично ограничивая себя требованием вычислимости. Вычислимость, о которой говорят конструктивисты, - воображаемая. Ведь все алгоритмы, которые рассматривают конструктивисты, существуют лишь в воображении конструктивистов. Выполнить эти алгоритмы в реальности невозможно - см. аргумент Вавилова о чернильной дыре. Так, в реальном мире невозможно построить два в степени гугол, несмотря на то, что вообразить алгоритм построения такого числа очень легко. Ибо для построения этого числа потребуется больше атомов, чем есть во всей вселенной. Претензии конструктивистов на якобы большую реалистичность по сравнению с математикой не имеют под собой почвы. Конструктивисты занимаются тем же самым воображением объектов, которым занимаются математики, однако почему-то кричат, что вовсе не пользуются никакими воображением.

У конструктивистов считается, что воображение не может быть связано с реальным миром. Конструктивисты заблуждаются. Например, если я возьму монетку и воображу её падение на землю, а потом действительно брошу монетку на землю, - я увижу почти то же самое, что я вообразил.

Для людей, которые стоят на позициях решительного утилитаризма и оценивают всякую деятельность постольку, поскольку она полезна в народном хозяйстве, также есть аргумент в защиту неограниченного воображения. А именно - математические теории можно рассматривать как черный ящик, как умозрительную машину, шестеренки которой приводятся в движение силой воображения; на вход подаются данные, на выходе получается предсказание. Никому не нужно, чтобы все воображаемые шестеренки такого калькулятора чему-либо соответствовали в реальном мире. Достаточно, чтобы он давал корректный результат при всех задачах, в которых ценное для утилитариста народное хозяйство нуждается. Если совершенно фантастические объекты верно предсказывают надои чугуна, то нет никаких причин требовать от этих объектов быть реальными. Математики занимаются созданием и обслуживаением фантастических шестеренок, строят в своём воображении конструкции, которые нужны лишь для других воображаемых конструкций. Каждый из математических объектов сам по себе для народного хозяйства бесполезен, однако из некоторых математических объектов в конце концов можно собрать очередную машину предсказаний. И нормальная математика справляется с этим куда лучше, чем конструктивизм. Достаточно указать на то, что все воображаемые шестеренки, лежащие под капотом классического анализа, в конструктивизме не существуют, - а вместе с этим невычислимый матан обусловил научную революцию и радикально изменил бытие человечества.

Математики, конечно, не видят смысл своей деятельности в создании калькуляторов. Математики в основном занимаются математикой, которая интересна внутри математики - употребляется в других разделах или даёт повод для воображения и изучения каких-то новых интересных сущностей. Полезные для народного хозяйства калькуляторы получаются непредсказуемым образом, лишь как случайный побочный продукт.
Аноним 07/06/17 Срд 09:28:24 19845 9
>>19669 (OP)
>народной мудрости /math
Первым делом, первым делом - матанализ
@
Ну а алгебра, а алгебра - потом.

Аноним 09/06/17 Птн 16:10:52 19985 10
>>19669 (OP)
Гусь, тебе никогда не казалось, что ты напрасно связываешь свою жизнь с математикой? Талантов у тебя нет, интереса — тоже, только зря насилуешь свой пролетарский мозг. А поступил бы в свой Ивановский политех — стал бы инженером. Или можешь разносить коробки с пиццей по домам. Или туалеты мыть — тоже хороший для тебя вариант. Я не шучу, просто подумай, математик хренов, где и кем ты работать-то будешь в перспективе...
Аноним 09/06/17 Птн 17:13:54 19990 11
Аноним 26/06/17 Пнд 15:13:52 21010 12
Градиент, дивергенция, ротор. Платина.

Градиент: показывает в какую сторону скалярное поле растёт быстрее всего, и с какой скоростью. Пример: поле - температура воздуха, градиент показывает куда надо из данной точки бежать, чтоб согреться.

Дивергенция: показывает, куда проёбывается хрень, переносимая векторным полем. Пример: двумерное векторное поле - скорость воды в неком мелком бассейне. Дивергенция будет показывать, где вода прибывает или убывает через дырки в полу.

Ротор: показывает области, вокруг которых векторное поле как бы крутится (сюрприз).
Аноним 28/06/17 Срд 13:22:31 21095 13
Мне интересно, откуда здесь столько физико- и програмисто- -быдла? Когда ещё маттреды были в /sci, то их было гораздо меньше, пара залетных. Сейчас всё наоборот. Физикопрогобыдла здесь большинство. Как так вышло? Зачем вы оккупировали нашу землю? Зачем вы, прямо как гитлеровские фашисты клеймившие евреев повязкой, клеймите нас пучкистами и тапало-гамологами? Убирайтесь от сюда! Вы только калечите землю своими примитивными инструментами! Я вижу, как вы отравляете реки, интегрируя по Риману, а не по Лебегу! Я помню сотни квадратных километров девственных, свежих лесов, но сейчас их нет, вы всё засадили картофелем! У меня страх за будущее поколение, когда вы, в своих конц. лагерях, нашим детям даёте определитель формулой, а не как эндоморфизм детерминантных векторов! Убирайтесь! ЗА БУРБАКОВ! ЗА ДЕДЕКИНДА! ЗА ЖАНА ЛЕРЕ!
Аноним 28/06/17 Срд 13:58:24 21100 14
Аксиома подстановки это эзотеризм!

Аксиома говорит вот что. Пусть ф - правильно построенная формула языка теории множеств первого порядка, в которую буква B не входит свободно. Пусть p - её строчка параметров. Тогда для любого множества A и любого набора параметров p если ф функциональна относительно A, то существует множество B, состоящее в точности из таких элементов, которые являются ф-образами элементов A.

Эта аксиома является эзотерической потому, что позволяет легко доказывать существование множеств, о пустоте или непустоте которых в принципе ничего сказать нельзя.

Например, правильно построенной формулой является формула "мощность x строго больше алеф-0, но строго меньше континуума". Применив эту формулу к какому-нибудь кардиналу большему, чем континуум, мы получим множество кардиналов M, пустота которого равносильна континуум-гипотезе. Множество M существует, но его элементы никакими средствами не могут быть изучены. Нелепость какая-то.

А ведь ф - любая правильно построенная формула. Которая к тому же может зависеть от любого числа параметров. Это, если вдуматься, позволяет буквально доказать существование Ктулху.

В самом деле, доказательство сводится к точному теоретико-множественному описанию того, что мы понимаем под Ктулху. Если C(x) - утверждение, что множество x является Ктулху, то может быть построена формула f(x, y), которая функциональна по x и каждому иксу сопоставляет Ктулху. Применив эту формулу к какому-нибудь одноэлементному множеству, мы докажем, что существует множество, единственным элементом которого является Ктулху.

Более того, если считать, что мы имеем право перенумеровать Великих Древних ординалами, то мы можем доказать существование любого наперед заданного количества экземпляров Ктулху. Даже алефа с трансфинитным нумером различных ктулх.
Аноним 05/07/17 Срд 02:53:33 21318 15
Объясните теорему Геделя о неполноте на пальцах.

Представим, что у нас есть формальная теория арифметики. В её алфавите N символов. Каждую букву алфавита этой теории будем считать цифрой, тогда каждая строка символов алфавита - натуральное число, записанное в N-ичной системе счисления. В частности, всякая формула - число.

Построим формулу F(x), в которой x - переменная. Скажем, что выводимость F(x) равносильна выводимости формулы, номером которой является число x.

Перенумеруем по порядку все формулы с одной переменной. Символом p|q обозначим формулу номер p, в которую подставили число q. Т.е. если обозначить номер формулы F как f, то f|14 означает всего-навсего F(14).

Пусть K - множество всех натуральных чисел k таких, что формула k|k невыводима.
Определим формулу G(x), выводимую тогда и только тогда, когда число x является элементом множества K. Номер формулы G(x) обозначим как g.

Если формула g|g выводима, то имеем формулу G(g). Значит, g является элементом K. Значит, g|g невыводима. Противоречие.

Если формула g|g невыводима, то g является элементом K. Значит, G(k) выводима. Значит, g|g выводима. Противоречие.
Аноним 26/07/17 Срд 16:20:43 22446 16
матфак - конвейер даунов
мехмат уже не торт
Куда бы ты не пошел, если не будешь
>нон-стоп ботать
ничего из тебя не получится.

А так вопрос тупой, конечно, если ты такими задаешься, не удосужившись даже погуглить, то дорога тебе на парашу, говно чистить, а не математикой заниматься.
Сейчас, наверное, на каждом углу трубят, что вышка=говно. Тем не менее, если грамотно набрать спецкурсов (а мб и ходить в НМУ), то можно и в говне стать хорошим математиком, но зачем вся эта морока, когда есть мехмат.
Аноним 27/07/17 Чтв 18:56:14 22500 17
>>19671
Не понял, а почему бы не считать составные типы категориями? Чем, скажем, List - не категория?
Допустим, тип Integer - это объект в категории Hask, тогда в List[Integer] - это он же в категории List, а fmap переводящий его из Hask на List - функтор. То же самое и с морфизмами, Типа, a -> b - морфизм на категории Hask, и есть функтор, такой, который переведёт его на категорию List, в виде List a -> List b.
Аноним 28/07/17 Птн 17:53:14 22528 18
Пони маете, ваша проблема в том, что вы не видите полной картины. Вы даже внятно не сможете объяснить зачем вообще нужны основания и аксиоматика и для чего там исчисление предикатов, например. С конструктивными основаниями в этом смысле все для вас еще хуже, все эти изоморфизмы карри'говарда, ВНК интерпретации, вычислимость, суждения, натуральная дедукция итд итп сами по себе требуют неких усилий для понимания, но еще хуже, что даже если отдаленное понимание присутствует, то все эти вещи не складываются в целую картинку,
тому що мозгов у вас як у хлебушка где все естественно вытекает одно из другого, как итог, конструктивная математика, а уж тем более MLTT для вас выглядит как какая'то лютая оккультная ебулда, бессмысленная и беспощадная, хотя по факту все там просто и понятно.
Аноним 08/08/17 Втр 12:47:22 22897 19
Как научиться доказывать теоремы

Тупой вопрос, и все равно не понятно, что ты от нас хочешь.
В любом случае прочитай книжку "Доказательства и опровержения" Лакатоса
Есть еще "Как решают нестандартные задачи" Канель-Белов, Ковальджи
У Пойи дохуя книг:
-Как решать задачу
-Математика и правдоподобные рассуждения
-Решение задач основные понятия, изучение и преподавание
Есть прекрасная классика
"Исследование психологии процесса изобретения в области математики" Жак Адамар
Наконец, "Как научиться решать задачи" Фридман, Турецкий и совсем простенькая "Как решать задачу, когда не знаешь как" (автора не помню)
Снизу я подчеркнул книги, в которых есть ответ на:
>Как понять, что нечто не очевидно нихуя, а надо как-то доказать?
Остальные - это прекрасные введения в эвристические задачи:
>Объясни мне, как научиться находить доказательства теорем
В том числе, в нескольких из них дают некоторые понятия, как классифицировать задачу, и, соответственно, какой подход будет наиболее рациональным:
>как научиться выводить самое удачное (или хоть одно, лол)
Сверху подчеркнул самые важные из них.
Есть еще сайт problems.ru - заходишь туда, ставишь легкий уровень и тебе автомат выдают серию задач, тренируешься, и потом переходишь на следующий уровень. В частности, на сайте собраны почти все классические олимпиадные задачи - идеально для тренировки эвристики.
Аноним 08/08/17 Втр 13:42:42 22899 20
Аноним 09/08/17 Срд 16:21:14 22951 21
Аноним 10/08/17 Чтв 11:54:59 22971 22
Аноним 11/08/17 Птн 21:32:34 23019 23
Аноним 30/11/17 Чтв 03:16:13 29421 24
Жан Лере, например, попав в плен, вынужден был скрывать свои знания и навыки, дабы не оказаться привлечённым к соответствующей работе. Даже если математик умеет решать диффуры, он в этом не признаётся.
Аноним 30/11/17 Чтв 06:03:46 29429 25
>>29421
His main work in topology was carried out while he was in a prisoner of war camp in Edelbach, Austria from 1940 to 1945. He concealed his expertise on differential equations, fearing that its connections with applied mathematics could lead him to be asked to do war work.
Аноним 01/12/17 Птн 00:31:36 29600 26
>>22500
>Чем, скажем, List - не категория?
Проиграл. То, что в императивном языке просто цепочка структур, связанных указателями, у хачкишкольников целая категория. Ну, чем бы девственники ни тешились, лишь бы Протопопова в /math не форсили.
Аноним 07/12/17 Чтв 23:41:32 30889 27
Подскажите, что в общем случае означает знак "="?


Категория состоит из объектов и морфизмов.
Морфизм — это нечто, что сопоставляет одно другому.
Возьмём категорию множества натуральных чисел. Каждым двум парам чисел соответствует их сумма. 2+2=4.
Равенство, суммирование — это морфизмы, определённые на категории.
Числа — это объекты.

Равенство двух категорий?
Морфизм между двумя категориями, называется функтором.
Т.е каждому морфизму и объекту одной категории, соответствует морфизмы и объекты другой категории.
Например, "*" соответствует "+". Числу " 1" соответствует "яблоко". Т.е, мы можем поставить соответствие над любыми объектами. Например, сиськи сопоставить лампочке, это тоже будет морфизмом.

Класс эквивалентности, это такой морфизм, на котором выполняется следующее:
1.Симметричность. A=B, тоже самое, что B=A
2.Рефлексивность. А=А. Объект равняется сам себе.
3.Транзитивность. Если А=Б и Б=С, то А=С
Если два вещи равняются третьей, то они равны между собой.

Функтор категорий является классом эквивалентности, тогда, когда выполняются эти условия.
Аноним 08/12/17 Птн 04:41:00 30911 28
>>19698
>бьющий мимо цели наезд на конструктивизм
Теперь понятно, кто постит в тред.
Аноним 08/12/17 Птн 14:35:28 30939 29
>>30889
Это предикатный символ в некоторой теории Т.
Обычно используется в инфиксная его запись.
На входе принимает два терма теории Т, на выходе даёт формулу теории Т.
Чисто синтаксическая вещь. Читай матлогику. Она, как и прочие содержательные теории, не зависит от выбранных оснований математики.
Аноним 09/12/17 Суб 09:17:13 31149 30
Вообще, это ваше чувственное отношение к противоречию, будто это что-то плохое - повергает меня в тихий ужас. Математики, блядь, незамутненная мысль. Если смотреть без эмоций, то противоречие - такой же надежный маркер, как и тавтология. Оно просто возникает в некоторых ситуациях, и на этой основе мы можем делать какие-то выводы. А уж какой это вывод - ну решать же только нам. Конечно, если вы любите, чтобы кто-то постарше решал всё за вас, то можно взять готовое решение в виде классической логики. Там противоречие - это ограда вокруг песочницы, "дальше детям нельзя".
А вы могли бы, например, работать в системе с игровой логикой, где несколько агентов ведут спор, и противоречие в ней - всего лишь техническое событие, как "переход хода".
Вы можете прямо сейчас создавать целые новые миры, вместо этого вы надрачиваете сморщенные хуи давно умерших людей.
Аноним 30/12/17 Суб 06:53:03 33479 31
Категория — это объекты, стрелки между ними. Последовательность стрелок[композиция], от одной вещи к другой. Существование у каждого объекта стрелки, которая возвращается. Ассоциативность.
Из этого можно выразить все, что угодно.

Множество — это соответствие одному слову список слов. Люди = {работники, учёные}

Пространство — соответствие каждому слову из списка, определённое количество других слов

Слово — это соответствие, название, обозначение чего-то чем-то.

Аноним 30/12/17 Суб 06:53:22 33480 32
Моноид - категория с одним объектом. Гомоморфизм моноидов - просто функтор между соответствующими категориями.

Кольцо - это линейная категория с одним объектом. Гомоморфизм колец - просто линейный функтор между соответствующими категориями.

Более общо, пусть R - коммутативное кольцо. Тогда R-алгебра - просто R-линейная категория с одним объектом, а гомоморфизм R-алгебр - просто R-линейный функтор.

Это даёт удобное определение модулей. Левый модуль над R - просто линейный функтор R -> Ab. Более общо, пусть R - произвольная линейная категория, когда C-модуль - просто линейный функтор C -> Ab. Гомоморфизмы C-модулей - естественные преобразования.
Аноним 30/12/17 Суб 18:23:35 33525 33
>>33333
Эти методы не используются в математике. Почему ты думаешь, что с помощью них можно доказывать что-то в математике?
Аноним 19/01/18 Птн 12:11:19 35254 34
На самом деле, платонизм, формализм и интуитивизм-конструктивизм - это, соответственно, математика невротика, математика психотика и математика аутиста (навязчивого невротика). Мысль о том, что вся математическая деятельность как таковая аутична по своей природе, и ей попросту нет места в остальных двух стилях мышления (которые, якобы, извращают её суть) звучит для меня очень неправдоподобно. И думается мне, что конструктивистов ещё озалупят не хуже, чем Гёдель озалупил формалистов. И это даст возможность поднять голову, скажем, платонизму, который переродится во что-то новое, сохранив при этом невротическую свою суть. Который потом также озалупят, дав возможность поднять голову, скажем, формализму, опять же, в перерождённой, но сохраняющей свою психотическую суть форме.

И так далее.

Математика реально развивается только посредством доказательств невозможности, которые и купируют нежизнеспособные идеологии, вскрывая их ограниченность.

И вопрос о том, что такое математика - это, по сути, вопрос о том, что такое человеческое мышление. И пока что открытым является в первую очередь именно вопрос о точной роли аутической компоненты человеческого мышления в математической деятельности - которая, на данный момент, явно сторонниками конструктивизма преувеличивается (небезосновательно, в конце концов, платонизм и формализм сейчас в идеологическом плане сидят под шконкой с фингалами под глазами).
Аноним 03/02/18 Суб 13:16:34 36306 35
Мой оптимизм основан на нескольких наблюдениях. Во-первых, теория категорий — сокровищница чрезвычайно полезных идей программирования. Haskell-программисты черпали из нее уже долгое время, и эти идеи медленно просачиваются в другие языки, но этот процесс идет слишком медленно. Нам нужно его ускорить.

Во-вторых, есть много различных видов математики, и все они предназначены для разных аудиторий. У вас может быть аллергия на математический анализ или алгебру, но это не означает, что вам не понравится теория категорий. Не побоюсь утверждать, что теория категорий — это именно тот вид математики, который особенно хорошо подходит для мышления программистов. Это потому, что теория категорий вместо того, чтобы иметь дело с деталями, оперирует структурой. Она оперирует такими понятиями, которые делают программы компонуемыми.

Композиция в самой основе теории категорий, она — часть самого определения категории. И я утверждаю, что композиция — суть программирования. Мы комбинировали вещи уже очень давно, задолго до того, как какой-то великий инженер придумал подпрограммы. Некоторое время назад принципы структурного программирования произвели революцию в программировании, — они сделали блоки кода комбинируемыми. Потом пришло объектно-ориентированное программирование, суть которого в комбинировании объектов. Функциональное программирование не только о комбинировании функций и алгебраических структур данных, еще оно делает параллелизм компонуемым, что практически невозможно с другими парадигмами.

В-третьих, у меня есть секретное оружие, нож мясника, которым я буду кромсать математику, чтобы сделать ее понятнее для программистов. Когда вы профессиональный математик, вы должны быть очень осторожны, чтобы определить все ваши предположения точно, выписать каждое выражение должным образом, и строить все свои доказательства строго. Это делает математические статьи и книги чрезвычайно трудными для чтения непосвященными. Я по образованию физик, и в физике мы добились удивительных успехов, используя неформальные рассуждения. Математики смеялись над дельта-функцией Дирака, которая была придумана великим физиком, П. А. М. Дираком, чтобы решить некоторые дифференциальные уравнения. Они перестали смеяться, когда придумали совершенно новую отрасль анализа, формализующую идеи Дирака и названую теорией распределений.
Аноним 03/02/18 Суб 17:02:30 36311 36
>>36306
>У вас может быть аллергия на математический анализ или алгебру, но это не означает, что вам не понравится теория категорий.
Все или ничего.
Либо ты не со мной, либо ты против меня.
Аноним 03/02/18 Суб 17:03:41 36312 37
>>36311.
Не заметил, что я в треде копипаст.
Аноним 03/02/18 Суб 17:04:00 36313 38
Аноним 03/02/18 Суб 18:47:47 36314 39
>>36306
>Во-первых, теория категорий — сокровищница чрезвычайно полезных идей программирования. Haskell-программисты черпали из нее уже долгое время, и эти идеи медленно просачиваются в другие языки, но этот процесс идет слишком медленно
Нахуй иди.
Аноним 05/02/18 Пнд 00:57:58 36371 40
Аноним 07/02/18 Срд 07:41:43 36425 41
>>36371
Всё таки, скорее ты. Он, конечно, грубо сказал, но Хаскелль действительно продвигать вредно и недопустимо.
Аноним 07/02/18 Срд 09:34:37 36428 42
>>36425
>но программирование действительно продвигать вредно и недопустимо.
фикс.
Аноним 09/02/18 Птн 21:29:53 36514 43
>>36428
>но теорию категорий действительно продвигать вредно и недопустимо.
Фикс фикса.
Аноним 10/02/18 Суб 00:20:29 36532 44
>>36514
>>36428

Пофиксил вам обоим за щеку! это интернет, детка

И программирование математики и теория категорий - отличные вещи. Конкретно язык программирования Хаскель никуда не годится.
10/02/18 Суб 00:47:43 36533 45
>>36532
>программирование
>язык программирования
Программирование и языки программирования действительно продвигать вредно и недопустимо.
Аноним 10/02/18 Суб 02:13:18 36535 46
>>36533
>Категории и теории категорий действительно продвигать вредно и недопустимо.
Так и есть.
Аноним 04/06/18 Пнд 16:03:51 40133 47
ПУЧК ПУЧК ПУЧК НИ ХОЧУ АНАЛИЗ УЧИТЬ ХОЧУТ АЛГЕБРАИЧЕСХУЮ ГАВНОМЕТРИЮ С ПУЧКАМИ ПУЧК И МНОГООООООБРАЗИЯМИ МАНЯФОЛЬДЫ МОИ СЛАДКИЕ РАССЛОЕНИЯ ВЕКТОРНЫ ВОТ ОНА АЛГЕБРА ТИОРИИ ТАМ ВСЯКИЕ А ЗАДАЧИ ЭТО ДЛЯ ИНЖИНЕРОВ ОНИ ТАМ ИНТРИГАЛЫ СЧИТАЮТ ДИФФУРЫ А МАТЕМАТИКА ОНА В ГОМОЕТРИИ КАТЕГОРИИ СЛАДКИЕ ПУЧК ПУЧК ПУЧК ВОТ ОНИ Я НИ ОДНОГО ИНТЕГРАЛА ЗА ЖИЗНЬ НЕ ВЗЯЛ А КТО ВЗЯЛ ТОТ ПРЕДАТЕЛЬ ВТОРОКУЛЬТУРНЫЙ И КАТОФАН С ВОДОФКОЙ ДЕДЫ ИНТЕГРАЛЫ РЕШАЮТ НИ МАТЕМАТИКА ЭТО ПУЧК АНАЛИЗ НА R НЕ НУЖОН ТАК Я СКАЗАЛ АКСИОМЫ ТАК ВПОЛНЕ УПОРЯДОЧЕННОЕ НЕПРЕРЫВНОЕ ПОЛЕ ВОТ ТАКОЕ У R ОПРЕДЕЛЕНИЕ А ДИДЫ МНОЖИСТВА СЧИТАЮТ НУ НИЧЕГО ВОТ КАТЕГОРИИ ПРИДУТ С МОРФИЗМЫ НАВЕДУТ
Аноним 04/06/18 Пнд 16:54:23 40135 48
>>36532
>Конкретно язык программирования Хаскель никуда не годится.
Для написания пейперов отлично годится. Еще для обучения алгоритмам. Короче, you have a difficult case of yaskozal.
Аноним 10/06/18 Вск 01:20:09 40303 49
Почему диды такие крутые?

У них есть все - свой дхду, свои люди в министерстве образования, физики их уважают, а гуманитарии им завидуют. От одного слова "матан" случайные прохожие разбегаются в благоговейном трепете, а при слове "пучки" люди только зовут модератора с формулировкой "несовершеннолетний". Пока неудачники кучкуются в каких-то подпольных клубах типа НМУ и носятся со своими листочками, деды возглавляют лучшие вузы страны. У них есть все: деньги, власть, уважение. Даже лоли у них есть. Про них снимают фильмы, а про тебя через 10 лет не вспомнит даже конструктивный петух.

Картофан - это успех. Деды - это сила. Война проиграна, господа.
Аноним 10/06/18 Вск 04:46:57 40305 50
>>40303
Войны никогда и не было. Нужна революция, пучковая весна!
Аноним 10/06/18 Вск 14:43:12 40314 51
>>40305
Доставьте ту вебмку про ватность.
Аноним 11/06/18 Пнд 18:50:02 40362 52
>>40305
А ну ка помацаем, кто у нас тут?.вебм
Аноним 22/06/18 Птн 19:05:42 40871 53

Using this metaphor, SEAR can be thought of as an ETCS-car which comes preassembled with a nice slick control panel. Or, using an alternate metaphor, ZFC is like Windows, ETCS is like UNIX, and SEAR is like OS X (or maybe Ubuntu). With SEAR you get a nice familiar interface with which it is easy to do standard things, there is less cruft than you get with ZFC, and behind the scenes you have all the power of ETCS (and more). (Of course, if you like Microsoft products, then this metaphor probably does not appeal to you.)
Аноним 23/06/18 Суб 13:28:48 40881 54
>>40871
Прежде чем учить ETCS и SEAR, надо ДО КОНЦА разобраться в более простых и классических вещах типа логики и теории множеств.
Аноним 23/06/18 Суб 13:37:06 40882 55
>>40881
Ого, а в клинописном письме и египетских иероглифах разобраться не надо? ETCS это и есть теория множеств. Учат её с нуля по Lawvere - Sets for mathematician.
Аноним 23/06/18 Суб 17:35:00 40892 56
>>40882

Дело в том, что так ты только примерную суть уловишь, а что-нибудь СТРОГО доказывать и записывать у тебя не получится.
Аноним 23/06/18 Суб 20:08:52 40914 57
Конструктивисты не получают новых результатов. Они просто раз в десятилетие берут какое-нибудь утверждение школьного уровня, например теорему Больцано-Коши, и пытаются передоказать конструктивно. Обычно ничего не получается.
Аноним 12/07/18 Чтв 18:29:50 41401 58
Почему сумма углов в треугольнике равна 180 градусам?

Я не знаю, зачем тебе это надо, свчешный ебанат, но так уж и быть.
>Мы все-таки в /матх, а не в /б.
Пиздец честь, долбоеб.

Аксиома принадлежности. Через любые две точки на плоскости можно провести прямую и притом только одну.
Аксиома порядка. Среди любых трёх точек, лежащих на прямой, есть не более одной точки, лежащей между двух других.
Аксиома конгруэнтности (равенства) отрезков и углов. Если два отрезка (угла) конгруэнтны третьему, то они конгруэнтны между собой.
Аксиома параллельных прямых. Через любую точку, лежащую вне прямой, можно провести другую прямую, параллельную данной, и притом только одну.
Аксиома непрерывности (аксиома Архимеда). Для любых двух отрезков AB и CD существует конечный набор точек A1, A2, …, An, лежащих на прямой AB, таких, что отрезки AA1, A1A2, …, An-1An конгруэнтны отрезку
CD, a точка B лежит между A и An .

Теорема 3.1.
Если внутренние накрест лежащие углы равны, то прямые параллельны.
Доказательство:
Докажем теорему так называемым методом от противного: предположим, что условие теоремы выполнено, а именно: прямые AB и CD образуют с секущей AC равные внутренние накрест лежащие углы, но вопреки утверждению теоремы прямая AB не параллельна прямой CD и, следовательно, они пересекаются в точке O, которая лежит в одной из полуплоскостей от прямой AC.
Отложим от луча АC треугольник AO1C, равный COА, так, что вершина O1 лежит в другой, нежели точка O, полуплоскости. Из равенства этих треугольников следует, что ^OAC = O1CA, ^OCA = ^O1AC; по условию: ^OAC = ^ACD и тогда точки O, C, O1 лежат на одной прямой, и, аналогично, из равенства по условию углов OCA и смежного к BAC следует, что точки O1, A, O лежат также на одной прямой. Отсюда следует, что через две различные точки O и O1 плоскости проходят две различные прямые AB и CD. Это противоречит аксиоме параллельных прямых. Полученное противоречие доказывает теорему.

Теорема 3.2.
Две прямые, параллельные третьей, параллельны.
Это свойство называется транзитивностью параллельности прямых.
Доказательство:
Пусть прямые a и b одновременно параллельны прямой c. Допустим, что a не параллельна b, тогда прямая a пересекается с прямой b в некоторой точке A, не лежащей на прямой c по условию. Следовательно, мы имеем две прямые a и b, проходящие через точку A, не лежащую на данной прямой c, и одновременно параллельные ей. Это противоречит аксиоме аксиоме параллельных прямых. Теорема доказана.

Теорема 3.4.
Если две параллельные прямые пересечены третьей прямой, то внутренние накрест лежащие углы равны.
Доказательство:
Пусть (AB) || (CD). Предположим, что ^ACD ≠ ^BAC. Через точку A проведем прямую AE так, что ^EAC = ^ACD. Но тогда по теореме 3.1 (AE) || (CD), а по условию – (AB) || (CD). В соответствии с теоремой 3.2 (AE) || (AB). Это противоречит аксиоме параллельных прямых, по которой через точку A, не лежащую на прямой CD, можно провести единственную прямую, параллельную ей. Теорема доказана.

Теорема о сумме углов треугольника — сумма углов треугольника на евклидовой плоскости равна 180°
Доказательство:
Пусть ABC' — произвольный треугольник. Проведём через вершину B прямую, параллельную прямой AC (такая прямая называется прямой Евклида). Отметим на ней точку D так, чтобы точки A и D лежали по разные стороны от прямой BC.Углы DBC и ACB равны как внутренние накрест лежащие, образованные секущей BC с параллельными прямыми AC и BD. Поэтому сумма углов треугольника при вершинах B и С равна углу ABD.Сумма всех трех углов треугольника равна сумме углов ABD и BAC. Так как эти углы внутренние односторонние для параллельных AC и BD при секущей AB, то их сумма равна 180°. Что и требовалось доказать.
Аноним 01/08/18 Срд 00:16:05 41736 59
>>19669 (OP)
Сделал содержание к лекциям по алгебре Вавилова на лекториум.тв:
https://pastebin.com/bznnRKL6
Наспех и коряво, но может пригодиться.

Легион схороняет и не забывает, но не наоборот!
Аноним 03/08/18 Птн 23:35:29 41799 60
>>41736
Как курс вообще?
Аноним 04/08/18 Суб 21:08:15 41813 61
>>22897
ДОКАЗАЛ ВПРАВО
@
ДОКАЗАЛ ВЛЕВО
Аноним 04/08/18 Суб 21:11:04 41814 62
>>41799
Вавилов затащил. Интересно рассказывает, сам вообще очень разносторонний, может пошутить, но часто использует еще не изученные понятия или отсылки к ним, а иногда вообще пытается описать универсальное свойство объекта, хотя до теории категории еще несколько семестров. .
Аноним 06/08/18 Пнд 10:15:57 41889 63
>>41736
Стоит смотреть?
Аноним 08/08/18 Срд 16:51:19 41965 64
>>19669 (OP)
из /un

Классический курс матана-I начинается с определения вещественных чисел по Дедекинду. Продолжается эпсилон-дельта определением предела последовательности, арифметических его свойств, свойств бесконечно-малых последовательностей и через них - свойств сходящихся. Дальше следуют теоремы Вейерштрасса, Коши-Кантора о вложенных отрезках, Больцано-Вейерштрасса о подпоследовательностях и критерий Коши. Потом даётся определение предела функции по Коши и по Гейне, доказывается их эквивалентность. С помощью предела по Гейне все теоремы о последовательностях переносятся на функции. Потом как-то вдруг возникают непрерывные функции и их свойства. Дальше идут друг за другом пафосные именные теоремы: Больцано-Коши, Вейерштрасса, Кантора, об обратной. Потом как-то сикось-накось определяются элементарные функции - традиционно проёбывается определение тригонометрических. Дальше всякие эквивалентные бесконечно-малые, типы неопределённостей и прочая туфта. Потом возникает производная. В этом месте препод, надувши щёки, важно возвещает, что производная-де - это тангенс угла наклона касательной. Затем впадает в своё обычное коматозно-горячечное состояние и выписывает таблицу производных. Дальше ещё одна порция именных теорем: Ферма, Ролля, Коши, Лагранжа, Тейлора, Лопиталя. Затем препод выписывает несколько рядов Маклорена, раскрывает несколько неопределённостей, исследует несколько функций, и на этом курс благополучно заканчивается. Впереди интегралы.

Так вот, физикам всё это не нужно. Ни один физик в своей деятельности эпсилон-дельта нотацией пользоваться сроду не будет. А у всех названных выше теорем не запомнит даже названий. Ну и зачем, спрашивается, огород городить?

Я предлагаю преподавать что-нибудь, что хотя бы как-то может пригодиться физикам. Начать можно с определения графа. Затем ввести функции как частный случай графа. Затем, таки да, определить категории и функторы. Не понимаю, почему все так их боятся, ведь определение категории через графы очень простое. Категория - ориентированный мультиграф такой, что
1. В каждой вершине висит петелька
2. Если есть путь из A в B, то есть и стрелка из A в B, соответствующая этому пути
Ещё нужно сказать пару слов о правилах манипулирования путями - по каким правилам пути можно приравнивать друг к другу, по каким правилам можно выкидывать из пути старые стрелки или добавлять новые. Совсем не сложно, правда? Уж всяко не сложнее, чем нудный рассказ о дедекиндовых сечениях.

Ориентированный - значит, каждое ребро является стрелочкой.
Мультиграф - между двумя вершинами может быть много стрелок, даже бесконечно-много. Каждая стрелка имеет своё собственное имя.
Путь - это последовательность стрелок, путь имеет вид A->B->...->Y->Z.

Располагая понятием категории, можно определить основные структуры pointless topology (тоже как граф), то есть очень быстро рассказать обо всём, что связано с непрерывностью. Потом ввести производные и интегралы как особую структуру в категории (и их тоже как граф, всё наглядно и никаких лишних больцано-кошей). Дальше - гладкие многообразия и классические структуры на них. И немедленно лагранжеву механику. Как, например, вот тут: https://arxiv.org/pdf/1612.03100.pdf

Таким образом, всю ключевую математику можно изложить не то что за один семестр, но даже за одну лекцию. В Ландау-Лифшице есть попытка сделать что-то похожее, но у Ландау не получилось, он пользовался слишком архаичными идеями.

Остаток семестра можно занять изучением какой-нибудь полезной теории когомологий (я бы предпочёл структуры Ходжа и когомологии Дольбо, но это не принципиально, можно и просто де Рама). А если останется время, то можно определить категории Фукая и рассказать теоретические основы M-теории. И всё это в первом семестре.

Преимущества такого пути очевидны. Физики не будут забивать себе голову бесполезными вещами, зато получат концептуально правильную интуицию и сразу же поймут, что же такое лагранжиан. Не просто услышат термин, как это часто бывает, но получат строгое и точное понимание, и даже немедленно смогут им пользоваться. Из некоторых недостатков - исчезает возможность шулерски прикидываться, что элементарные функции определены. Но на самом деле это не недостаток, и вот почему:

Давать физикам строгое определение элементарных функций бесполезно. Оно требует очень искусного определения вещественных и комплексных чисел, а физики не изучают даже строгую теорию вещественных чисел. Для неё требуется продвинутая теория множеств, а у физиков нет времени на теорию множеств. Как правило, физик, проучившийся своему "матану" целых два года, даже не сможет внятно рассказать, почему 0.(9) = 1, и начинает лепетать что-то невнятное про какие-то там бесконечно-малые. Не говоря уже о более хитрых вопросах - например, почему класс интегрируемых по Риману функций шире класса непрерывных функций, т.е. из-за каких особенностей определения интеграла Римана такое произошло, т.е. какова же причина справедливости критерия Лебега. А ведь по бумагам физик должен знать такие вещи. Бумаги, таким образом, лгут.

Поэтому считать производные, брать интегралы, манипулировать рядами, жонглировать множителями Лагранжа - в общем, всем рутинным вычислениям нужно учить без глубокой теории, чисто механически. Так же, как делению в столбик и вычислению определителей методом Гаусса. И делать это нужно на семинарах, а не на лекциях. Тупо выдать таблицу производных и научить ею пользоваться; де-факто так и происходит.

В общем, я считаю, что тратить время на бессмысленное повторение никому не нужных вещей попросту нелепо. Но пока в университетах преподают старые маразматики пенсионного возраста, из года в год талдычащие одну и ту же архаику, хороших вещей у нас не будет.
Аноним 08/08/18 Срд 17:32:12 41966 65
>>41965
Ну и маразматичный же текст (паста?). Как ты собрался говорить о гладких многообразиях не используя вещественных чисел? Когомологии Дольбо — окей, комплексные числа — слишком сложно? Таблица интегралов для запоминания, ибо вывод (уровня детсада) слишком сложен? И т. д.

Правда, согласен с тем, что присутствующая в начале типичного курса анализа и преподносимая с невероятным ампломбом и пафосом последовательность тавтологических переформулировок аксиом, гордо именуемых «теоремами» и общая законсервированность и маразматичность священного курса великого матанализа раздражают. Но это общеобразовательная проблема, а не математическая.
Аноним 08/08/18 Срд 18:09:04 41967 66
>>41966
Поддвачну. Какой то реально поехавший писал. Физикам не нужон этот ваш эпсилон-дельта, давайте лучше теорката хуйнем. Лолшто блядь?
Аноним 08/08/18 Срд 18:26:53 41970 67
>>41966
Речь не о том, чтобы не использовать вещественные числа; речь о том, чтобы не пытаться впихнуть в физиков их строгое построение. Можно пользоваться R^n, не зная всех логических тонкостей теории вещественных. Вывод таблицы интегралов действительно сложен - для полноценного её обоснования, не прибегающего к размахиванию руками, необходима обстоятельно разработанная теория аналитических функций, начиная с формулы Коши-Адамара. Ну а нестрогий вывод есть бесполезное насилие над студентом, который в результате проебет кучу времени, но на самом деле ничему не научится. Основные идеи мейнстримных теорий когомологий понять ничуть не труднее, чем запомнить всё то, чем пичкают студентов, обучая их "интегрированию".

>>41967
Весь необходимый для лагранжианов теоркат прекрасно описывается графами. Определение категории я уже очертил, например, - ты его не понял? Графы гораздо проще эпсилон-дельта. И интуитивнее. Ты думаешь, физику эпсилон-дельта нужнее лагранжианов?
Аноним 08/08/18 Срд 18:28:45 41971 68
>>41970
О, а вот и автор пришёл. Так и знал, что это кто-то из /math
Аноним 08/08/18 Срд 20:52:08 41975 69
>>41970
Весь необходимый для лагранжианов теоркат прекрасно описывается следующим предложением. .
Аноним 08/08/18 Срд 21:48:39 41976 70
Аноним 08/08/18 Срд 22:03:58 41977 71
>>41976
Вообще это такая охуительная дихотомия
> Ты думаешь, физику эпсилон-дельта нужнее лагранжианов?
что при первом прочтении, честно скажу, она как то прошла мимо меня. Ты видимо подразумеваешь что чтобы объяснить физику что такого лагранжиан нужно сначала объяснить ему теорию категорий. Я правильно понимаю? Ээээ... и нахуя? Жду объяснений. На уровне определений и решения простых задачек знаком и с тем и с другим.
Аноним 08/08/18 Срд 23:38:59 41978 72
CB32EF64-DC2F-4[...].jpeg 81Кб, 1024x768
1024x768
>>41970
>Речь не о том, чтобы не использовать вещественные числа; речь о том, чтобы не пытаться впихнуть в физиков их строгое построение. Можно пользоваться R^n, не зная всех логических тонкостей теории вещественных.

Ок, мысль ясна. В принципе так можно.
Но, кстати, что тут:
Определение: полное линейно упорядоченное поле.
Конструкция: пополнение рациональных чисел.
сложного?

>Вывод таблицы интегралов действительно сложен - для полноценного её обоснования, не прибегающего к размахиванию руками, необходима обстоятельно разработанная теория аналитических функций, начиная с формулы Коши-Адамара. Ну а нестрогий вывод есть бесполезное насилие над студентом, который в результате проебет кучу времени, но на самом деле ничему не научится.

Формула Коши-Адамара (элементарнейшее соображение сравнения с геометрической прогрессией, применяющееся повсюду), это, конечно, ебать как сложно.

Таблица интегралов — это пикрелейтед (первая выдача google по соответствующему запросу)?

Аналитические функции — это чтобы опеределить синусы/косинусы/экспоненту?

Вообще, я ничерта не понимаю. Только что ты говорил: нахуй строгость, все и так знают, что такое R. А потом такой: с синусами только строго, только ряды, иначе бессмысленное насилие и зашквар. WTF?

>Основные идеи мейнстримных теорий когомологий понять ничуть не труднее, чем запомнить всё то, чем пичкают студентов, обучая их "интегрированию".

Когда «пичкают приёмами интегрирования» (?) понимать не надо, надо считать. Некоторые (многие) студенты это любят.
Когомологии — из другой оперы. Я согласен с тем, что ничего невероятно трудного тут нет, но, простите, когомологии де Рама требуют кучи предварительных определений. Внешние степени / дифф. формы, сами многообразия, комплексы, ядра/коядра... Такие вещи (теорема Стокса, когомологии де Рама и т.д.) бывают в курсах, но в конце, а не в начале. А когомологии вообще — огромная тема, которая скорее уместна на топологии, чем на анализе.

>Весь необходимый для лагранжианов теоркат прекрасно описывается графами. Определение категории я уже очертил, например, - ты его не понял? Графы гораздо проще эпсилон-дельта. И интуитивнее. Ты думаешь, физику эпсилон-дельта нужнее лагранжианов?

Бегло просмотрел текст по твоей ссылке , более-менее стандартное изложение, куча формул. Хотя в теме классической механики я и не очень разбираюсь, но всё равно, при чём тут теоркат действительно не вижу.
В определение производных и интегралов с помощью категорий верю (кстати, где оно?), а вот в то, что оно реально превносит что-то новое и упрощает понимание — не очень.
Аноним 09/08/18 Чтв 02:28:03 41984 73
>>41970
Теоркат через графы... ну в общем насрать, физики привыкли говно жрать.
Аноним 09/08/18 Чтв 03:24:38 41985 74
>>41984
Вполне здраво же
Аноним 09/08/18 Чтв 05:12:31 41986 75
>>41985
Нет. Категории нужно изучать вместе с фундаментальной группой.
Изучать что-то само по себе, если ты не аутист, сложно.
Аноним 10/08/18 Птн 17:49:01 42026 76
>>41986
1) какая связь между теоркатом и фундаментальными группами?
2) нахуя нужны эти твои фундаментальные группы физику и вообще?
Аноним 14/08/18 Втр 07:06:30 42131 77
>>42026
>1) какая связь между теоркатом и фундаментальными группами?
Пиздос! Они вообще-то исторически появились от туда.
>2) нахуя нужны эти твои фундаментальные группы физику и вообще?
нахуя нужны эти твои физики и вообще?
Аноним 14/08/18 Втр 12:13:48 42132 78
>>42131
Чтобы делать мир лучше, пока маняматики сидят на грантах и доказывают нинужныя теоремы?
Аноним 14/08/18 Втр 13:45:51 42133 79
>>42131
Ну а векторы появились как название для невещественной части кватернионов, давайте теперь вместо аксиоматической теории векторных пространств изучать кватернионы. Изучать что-то в его историческом развитии - как правило, худший способ изучать это что-то.
Аноним 16/08/18 Чтв 15:37:21 42176 80
>>41965
>И немедленно лагранжеву механику.
И немедленно выпил.
Аноним 19/08/18 Вск 12:43:43 42239 81
>>29421
Если бы я умел брать интегралы, то в этом тоже не признаюсь.
Аноним 19/12/18 Срд 19:16:57 47342 82
Полезно подходить к теоремам следующим образом. Сначала предположить, почему утверждение вообще должно быть верно - на пальцах/интуитивном уровне (не всегда работает по началу). Потом подумать, а как вообще можно доказать. Потом пробежать доказательство и составить его план, опуская технические детали - спустить его до интуитивного уровня. Потом подумать, почему твоё доказательство не сработало бы. Ну и проделать все шаги самому.
Аноним 09/10/19 Срд 16:28:14 59771 83
>>19669 (OP)
>золотые россыпи крупиц народной мудрости /math
Если долго пучкаться, что-нибудь получиться!
Аноним 09/10/19 Срд 16:28:59 59772 84
Влияние Додика на возникновение теоретико-категорной точки зрения выясняется в этой статье:
https://pdfs.semanticscholar.org/ee5c/a5d8de2d3ab82dbe1ef2817c20f201877f51.pdf
Первую и вторую культуру определил Тимоти Гауэрс (отнеся себя ко второй). Это статья сама по себе является отсылкой к статье Ч.П. Сноу (тот самый писатель который еще предисловие к "апологии математика" Харди написал).
В статье Гауэрса это не очень ясно, я придумал следующее упрощение: conceptual theory building vs concerete problem solving.
Но это не совсем точно. И. М. Гельфанд говорил что надо решать задачи, а не строить теории, тем не менее он однозначный первокультурщик. Серр тоже предпочитал задачи теориям, см. его переписку с Гротендиком.
Можно так же сказать, что в концептуальной математике принято улучшать доказательства, например доказательство Хопфа обобщенной формулы Эйлера-Пуанкаре не использовало гомологических методов и было очень громоздким.
Еще один аспект улучшение языка. Кардано решал кубические уравнения геометрическими построениями, современный символьный язык алгебры получил распространение поздеее, у коссистов. Ясно пользоваться символьной нотацией удобнее. Уравнения Максвелла на языке дифференциальных форм выглядят гораздо проще, тогда их всего два, а не 8.
Язык производных функторов позволяет высказывать утверждения, которые пользуясь спектральными последовательностями, записать невозможно. Вместе с тем всегда можно перейти к явным вычислениям со спектралками. Как от символьной записи квадратного уравнения можно перейти к построениям Кардано, если нужно.
Так же можно сказать о строгости. В комбинаторике, классическом анализе, можно получать крупные результаты пользуясь нестрогими методами, например Эйлер, Рамануджан.
Бескоординатый язык концептуальнее даже в физической ситуации, потому что не надо доказывать независимость от координат. Интеграл Лебега лучше интеграла Римана, потому что пр-во интегрируемых решений полно и значит не нужно доказывать существование решений.

Короче, перефразирую Посицельского. Мы хотим иметь простые доказательства. Это достигается за счёт сложных определений.
Аноним 15/10/19 Втр 16:04:54 59999 85
>>59771
Сколько селедку пучками не корми - она все на интегралы смотрит...
Аноним 23/03/20 Пнд 13:19:40 66517 86
>>19694
>Математика - это заложенный самим Богом
Дальше не читал. Религиозных уебков надо изгонять, а ваши храмы упразднить. Сколько денег народа тратится на эти ебучие храмы. Мы могли бы лучше жить, если бы не надо было кормить храмы и попов.
Аноним 23/03/20 Пнд 13:25:22 66518 87
Аноним 15/05/20 Птн 01:11:39 68832 88
Смотри тут всё просто: теория множеств, все с ней ебались как дети слепые,пока не сказали, ой блядь, да смотрите же из декартовой замкнутости можно всё вытащить. Вкинули в эту хуйню еще топологию, полочули универсальную модель всей теории топологии, компактов, там блядь даже лямбда куб баренрехта живет.

Потом гомологической алгеброй увлекались после войны. Сказали а чё пацаны если не множества а любая размерность, любые группы, полная свобода в любом направлении до бесконечности. Ок построили несколько категорных моделй, одна из которых триангулируемые категории.

Потом пришел Квиленн и сказал это все хуйня, надо что-то гораздо глубже, нужно показать нахуй связь всех дуальностей и родил новые основания математики модельные категории Квиллена и гомотопическую алгебру. Тут блядь понеслось. Оказалось у категории множеств (топос) ровно девять моделей Квиллена, Топологии две: на фибрациях Гуревича и Серра. После этого все стали в Квиллен категориях работать.

Пришел Воеводсткий, посмотрел на Кок и сказал о так охуенно тут же можно прямо CW-комплексы выражать. Ну а маметики топовые чем отличаются от обычных, тем что сразу основания давай цементировать и воду в унитазы спускать заливая всеговном. Воеводский взял сначала симпилиальные множества (как предпучки на категориях симплексов со значениями в категории множеств) -- соснули хуйца, потом сказали ок, ну кубической точно хватит, расчехлили стары пейперы и полетели. В 2016 окончательно построили Квиллен категорию для кубической теории и классифицирующий топос рассмотрели, доказали каноничность --- это сейчас фронтир современных симплициальных оснований в которых уже можно "свободно дышать".

Но математики не хакеры, имы движет желание пробросить слона сквозь замочную скважину. Вот возьмем опять Воедоский например для того чтобы простроить доказательство простого уравнения из когомологической алгебры он построил целут теорию HoTT-A^1 условно будем ее называть. Прикинь, создал целый предмет, как Эйленберг, Квиллен, Гротендик. Вот за такой проброс слона через хуй и дают Филдса, и это тогда настоящая математика.
Аноним 15/05/20 Птн 12:28:23 68862 89
А на пикриле пучок?
Аноним 15/05/20 Птн 22:13:53 68881 90
Аноним 23/06/20 Втр 20:36:39 70495 91
Читать теорфизику абсолютно бесполезо, что по сасскинду что по ЛЛ, если не знаешь общую физику. У нас таких было полпотока - про лагранжианы знают, фазовые портреты, алгебры наблюдаемых и скобки пуассона, хуёмоё, а собственно реальной физики не знают. Но от целей конечно зависит
Аноним 23/06/20 Втр 21:12:34 70497 92
>>70495
Прямо меня описал
Аноним 25/06/20 Чтв 08:32:37 70547 93
пучок.png 400Кб, 1200x1679
1200x1679
Аноним 25/06/20 Чтв 09:43:58 70552 94
>>70547
красивая картинка, что за книжка?
Аноним 25/06/20 Чтв 09:48:25 70554 95
>>70552
Любовь и математика Эдвард Френкель.
Аноним 25/06/20 Чтв 11:27:39 70559 96
>>70554
спасибо, кажется, хорошая
Аноним 25/06/20 Чтв 22:24:17 70578 97
ПУЧКИ.png 112Кб, 291x410
291x410
>>70547
>Вот пучок
${\color{Red} {ПУУУУУУУУУУУУЧЧЧЧЧЧЧЧЧЧЧЧКККККККККККККК}}$
26/06/20 Птн 00:35:52 70581 98
>>70578
Впилили им тех, пиши формулы, нет, не хочу формулы писать, хочу сМиЩнЫе НаДпИсИ разноцветные делать. Пиздец.
Аноним 26/06/20 Птн 00:53:29 70582 99
>>70547
Насмотрелся на этого вашего Пыньку и тоже захотел понять наконец что же это такое ПУЧОК. Но пока что сколько над определением ни медитировал никак не заходит. И что то там нихуя про векторные пространства я не вижу. Может посоветуете что нибудь максимально доступное чтобы обпучкаться наконец?
Аноним 26/06/20 Птн 03:56:30 70583 100
>>70582
>И что то там нихуя про векторные пространства я не вижу.
Там каждому открытому множеству U топ. пространства сопоставляется множество F(U). Это множество F(U) может быть чем угодно, хоть просто множеством, хоть группой, хоть векторным пространством...
>что нибудь максимально доступное
Ramanan Global Calculus
Аноним 26/06/20 Птн 03:57:54 70584 101
>>70582
ключевое слово -- "Окольцованное пространство" (ringed space)

без мотивации, толкающей тебя в алгебраическую/комплексную геометрию, едва ли это стоит изучать. в анализе, правда, оно используется для доказательства теоремы де Рама, но больше почти нигде, в сущности Соответственно, ты спрашиваешь про хорошие пособия по алгебраической/комплексной геометрии для новичков. Это, наверное, подскажут.

щпределение на самом деле не особо трудное, так что читать его можно практически везде. (к тому же, написано и повторено оно бесчисленное число раз)

Чисто по пучкам были лекции Львовского где-то из позапрошлых веков.
Аноним 26/06/20 Птн 11:32:00 70587 102
>>70583
Только на картинке написано что каждой точке ставится в соответствие линейное пространство. Ты либо реально тупой уебан что не видишь никакой разницы между тем что там написано и тем что ты написал, либо на столько просветленный что для тебя нет никакой разницы, уж не знаю.
>>70584
>хорошие пособия по алгебраической/комплексной геометрии для новичков
Давай, гляну
>лекции Львовского
К Львовскому отношусь мягко говоря насторожено после того как наткнулся на задачку Рамануджана в его лекциях по матану.
Аноним 26/06/20 Птн 11:37:05 70588 103
>>70584
>к тому же, написано и повторено оно бесчисленное число раз
Если написать одну и туже хуйню много раз она становится понятней. Кек.
Если написать одну и туже хуйню много раз она становится понятней. Кек.
Если написать одну и туже хуйню много раз она становится понятней. Кек.
Аноним 26/06/20 Птн 15:47:58 70593 104
>>70587
>Только на картинке написано что каждой точке ставится в соответствие линейное пространство.

я вот не совсем хорошо понимаю, что там автор имел в виду, однако по аналогии с расслоениями, возможно, речь о том, что если у тебя над каждой точкой висит векторное пространство, которое гладко зависит от точки, то локально конструкция изморфна прямому произведению векторного пространства и окрестности точки (тогда получается определение >>70583); обратно, если взять определение >>70583, то каждой точке можно сопоставить пространство, которое соответствует окрестности этой точки.

пространства, которые висят над точкой и гладко меняются в зависимости от точки в точности порождают векторное расслоение, они же порождают и т.н. локально-свободный пучок; эти две конструкции эквивалентны

определение >>70583, по модулю деталей, правильное, конечно правда, строго говоря, оно ближе к понятию предпучок, но не суть


львовский хороший, записки указанные тоже.
но без сильной мотивации читать их почти невозможно (в них много упражнений к тому же). я бы рекомендовал читать про геометрию всё же.

слайды вербицкого можно (любые по любой геометрии, у него пучки везде, кажется)
Аноним 26/06/20 Птн 17:18:47 70598 105
>>70588
если почитать одно и то же в разных источниках, оно действительно может стать понятней

ну, а имбицилам ничего не поможет
Аноним 01/09/20 Втр 04:22:09 72571 106
В обыденном языке мотивацией называется побуждение другого человека (или себя самого) к выполнению некоторого действия. На математической фене "мотивация" - это форма самоотчета автора: автор сообщает читателю, по какой причине и ради какой цели излагается данный материал, т. е. какие дидактические соображения мотивировали автора построить изложение предмета тем или иным образом.

Дело в том, что математическое знание оформляется (да и создается, пожалуй, тоже) не абы как, а по некоторым лекалам (метапаттернам). Из-за когнитивных ограничений, человек не способен охватить весь предмет целиком, поэтому метапаттерны нужны для организованного разбиения крупного массива знания на более мелкие и легкоусвояемые части, своего рода "архитектурные блоки", которые человеческий мозг может легко обработать, понять и разложить на базовые интуиции. Эти блоки могут быть разными, и они могут по-разному связаны друг с другом - в результате один и тот же раздел математики может быть изложен множеством различных способов. Выбранный способ изложения предмета - это своего рода "архитектурное решение": как и в настоящей архитектуре, оно может быть удачным или неудачным, удобным или неудобным, полезным или бесполезным, эстетически привлекательным или отвратительным.

Вообще говоря, где-то на стыке математики, психологии и педагогики уже давно должен был возникнуть особый предмет исследования под названием "Архитектура Математики" - но т. к. психологи математику знают плохо, а математики участвовать в психологических экспериментах особо не рвутся, то шансы на то, что подобный предмет появится если не сейчас, то хотя бы в ближайшем будущем, довольно малы. Поэтому авторы современных учебников, точно так же как и дедушка Евклид 2000 лет назад, вынуждены заниматься кустарщиной, полагаясь исключительно на чуйку, актуальные тренды и личный опыт преподавания. В итоге мы имеем сотни омерзительных, перегруженных, запутанных и абсолютно нечитабельных компендиумов и компилятов черт пойми чего, продающихся под видом учебников математики.

Вот к каким печальным результатам приводит отсутствие мотивации - как обычной, так и математической.
Аноним 07/09/20 Пнд 15:25:31 72824 107
Для начала нужно перестать ломаться аки девица и затопырить брезгливо оттопыренную губу. Даже школьная алгебра иногда бывает достаточно груба, чтобы спустить зарвавшегося зазнайку с небес на землю. Выше по треду уже ползает парочка таких сбитых дистрибутивностью мересьевых - умом громам повелевали, но треснули пополам в тщетной попытке умножить икс на одну вторую.

Поэтому, снизив уровень притязаний и упырив мел, снова садимся за парту. Скачиваем и читаем книги Гельфанда в указанном порядке: "Метод координат", "Функции и графики", "Алгебра". Решаем минимум 80% задач из них (это где-то 300 задач в совокупности). Геометрией и тригонометрией позволительно пренебречь.
Параллельно читаем книги Пойа, или хотя бы упрощенную и адаптированную для советских школьников версию его единственно верного учения - например, книгу Фридмана и Турецкого, "Как научиться решать задачи".

Завершив сию подготовительную процедуру, идем в ученики к мастеру Шеню. А уж дальше как бог даст. Несмотря на свою почти детскую простоту, Шень оказывается не по зубам 90% населения, и редкая девица дочапает хотя бы до его середины. А уж до боссов первого уровня, вроде Кострикина, добирается едва ли один из ста игроков.

Глупые белые девки с катанами, сколько вас таких по весне оттаяло.
Аноним 07/09/20 Пнд 16:36:31 72838 108
>>72824
>книги Гельфанда в указанном порядке: "Метод координат", "Функции и графики", "Алгебра"
почему такой ебанутый порядок?
Аноним 07/09/20 Пнд 17:00:50 72840 109
>>72838
не знаю, я ни одну из этих книг не читал
Аноним 27/09/20 Вск 12:53:38 73831 110
программа наивной теории множеств должна быть устроена так:

Элементарная логика
высказывание
таблицы истинности для и, или, не, импликации
предикаты

Множество
элемент, включение, подмножество, равенство, пустое множество
схема выделения
булеан (множество всех подмножеств)
конечные объединение, пересечение, разность
диаграммы Эйлера
неупорядоченная и упорядоченная пара (по Куратовскому)
упорядоченная n-ка, конечные произведения
графическое истолкование предикатов

Отношения
бинарные и n-арные
эквивалентности, фактормножества

Отображение
график отображения
образ, прообраз, слои, обратное отображение, связь с факторизацией
сюръекция, инъекция, биекция (критерий), "равномощность"
семейства множеств
бесконечные объединения, пересечения
функция выбора, бесконечные произведения
принцип де Моргана
характеристические (индикаторные) функции
их связь с теоретико-множественными операциями
y^x как множество всех отображений из x в y
2^x как булеан x

Счетное множество
произведение двух счетных множеств счетно
объединение счетного семейства счетных множеств счетно
N, 2N, Z, Q, QxQ и Q^n счетны
семейство попарно не пересекающихся интервалов не более чем счётно;
семейство попарно не пересекающихся окружностей на плоскости, касающихся оси абсцисс, не более чем счетно
семейство попарно не пересекающихся шаров в R^n не более чем счетно
множество алгебаических чисел счётно
множество точек разрыва монотонной функции счётно
отрезок несчетен
2^N несчётно

Фракталы
канторово множество, двоично-рациональные числа, троичная система
бесконечные деревья
треугольник и ковер Серпинского, губка Менгера
картинка для кривой Пеано

Континуум
интервал, отрезок, 2^N и R равномощны
Мощность c имеют:
график функции sin
квадрат, круг, окружность на плоскости
сфера, шар, куб в R^n
множество R^n
булеан счетного множества
множество Кантора
объединение континуального семейства счетных множеств

Теорема Кантора
о биекции множества и его булеана
Парадокс Кантора
Теорема Кантора-Бернштейна-Шрёдера

Порядок
частичный, линейный, полный, точные грани
диаграммы Хассе
аксиома выбора, лемма Цорна, теорема Цермело, принцип Хаусдорфа

Порядковые типы
канторовские порядковые типы
арифметика порядковых типов
омега и обратный омега (w и w^op),
w+1, w+w, 2^w, w^w, w^w^w^... (эпсилон-нуль)
эта (рациональные числа)
лямбда (вещественные числа)
критерий вуп (вполне упор. множества) - не содержит подмножества типа w^op.
порядковый морфизм, изоморфизм
если f порядковый эндоморфизм полного порядка, то x <= f(x)
начальный отрезок, отсутствие биекции вуп на свой начальный отрезок
два начальных отрезка не изоморфны
между вуп может быть не более одного изоморфизма

Ординалы
натуральные числа по фон Нейману-Цермело
порядок на ординалах
любое множество ординалов вполне упорядочено
парадокс Бурали-Форти
теорема Хартогса, пределы
арифметика ординалов, нормальная форма Кантора
трансфинитная индукция и рекурсия, примеры
кумулятивная иерархия V Цермело - фон Неймана
конфинальность, омега-первое, мощности алеф-0 и алеф-1
отсутствие промежуточных между алеф-0 и алеф-1
алеф-1 не больше континуума, континуум-гипотеза

Кардиналы
аксиома подстановки (аксиома Френкеля)
числовые классы, омега-n для любого ординала n
числа алеф, числа бет и числа гимель для любого ординала
тип эпсилон-n для любого ординала n
арифметика кардиналов
цепные дроби, факториалы, корни и логарифмы кардиналов
теорема Кёнига, следствия о мощностях 2^алеф0, алеф0^алеф0, c^алеф0 и т.п.
регулярные и сингулярные кардиналы, недостижимые кардиналы (определение)
Аноним 27/09/20 Вск 13:43:47 73836 111
>>73831
уже было здесь

ни один приличный человек в здравом уме учить "неаивную теорию множеств" по отдельной программе не будет. потому и программа эта не нужна, неважно, как она "должна быть устроена"
Аноним 27/09/20 Вск 15:39:18 73841 112
>>73836
Чтобы учить аксиоматическую теорию, нужно знать "наивные" идеи
"наивная" - стремное название вообще, правильное - "канторовская"
Аноним 27/09/20 Вск 19:57:56 73866 113
>>73836
>уже было здесь
Да ладно? Тред копипасты как раз и для скидывания сюда мудрости.
Аноним 27/09/20 Вск 23:27:29 73870 114
>>73836
Будут вообще: https://www.youtube.com/watch?v=PmvIytQ24HM но кстати странный аргумент, в моё время когда я учился в первом семестре было два отдельных курса: "логика" и "дискретная математика", и по "логике" рассказывали (полу)аксиоматическую теорию множеств с аксиомами и примерно по этой программе. Ну там расписана всё страшно но на каждую строчку потратить 5-10 минут можно.
Аноним 28/09/20 Пнд 02:12:54 73871 115
>>73866
а, ну тогда ок
на доске так редко появляется что-то новое, я в название тредов не смотрю

>>73841
не знаю, для чего её учить
все эти "наивные" быстро идеи постигаются по мере изучения нормальной математики
из приведённой программы мне незнакомы только последние две главы (про ординалы и порядок), возможно, потому, что это нигде не используется подскажи, где, если знаешь
Аноним 28/09/20 Пнд 02:27:35 73872 116
>>73871
Лемма Кёнинга используются много где, обычно в конструкциях когда берут всё более мелкие и мелкие разбиения чего нибудь и хочется показать что "предел разбиений это точка". Ну скажем в теореме о том, что комплекс сингулярных цепей представимых как сумма из цепи из множества $U$ и множества $V$ квазиизоморфен комплексу сингулярных цепей на $U \cup V$ (последовательность Майера-Виеториса).

Всякие штуки связанные с большими множествами (парадокс Буралли-Форти и т.д.) нужны для аккуратной работы с большими категориями, ну скажем чтобы понимать что если кополная категория имеет все копределы по большим диаграммам, то она частичный порядок.

Маленькими счётными ординалами часто естественно нумеруются всякие инварианты всяких алгебраических объектов (нижний центральный ряд, производный ряд, аугментационная фильтровка) что иногда бывает полезно для маломерной топологии (запредельная алгебра и топология Михайлова та же). Ну не говоря уже о приложениях в логике: ординалы доказуемости, нестандартный анализ, дескриптивная теория множеств, инварианты континуума (ib4: это не математика).

Ну моё мнение что математик всё это кое-как знать на каком-то этапе всё же должен, но в принципе учить отдельно смысла нету наверное (ну кроме леммы кёнинга, её всё же знать нужно всем).
Аноним 28/09/20 Пнд 09:05:52 73874 117
>>73871
>не знаю, для чего её учить
Во-первых, это прикольно.
Во-вторых, это лёгкие вещи. У нормального математика (который знает лемму Цорна) на всё остальное уйдёт максимум пара часов, и ещё сколько-то на упражнения. Спокойно можно всё заботать в метро или в туалете.
Аноним 28/09/20 Пнд 09:17:01 73875 118
>>73874
>Во-первых, это прикольно.
Ну, если совсем нечем заняться
Аноним 28/09/20 Пнд 12:03:08 73878 119
>>73874
>Во-первых, это прикольно
Скучаю по тем временам, когда у меня было достаточно вреемени для занятия всякой хуйнёй только из-за того, что это "прикольно".
А так-то аксиоматическая теория множеств - анально огороженная хуйня без задач в 99.999% современной математики.
Аноним 28/09/20 Пнд 19:33:49 73895 120
>>73878
Речь не про аксиоматическую
Аноним 14/10/20 Срд 12:56:20 74716 121
Не выучивай, осознавай. Представь, что ты кот, и у тебя есть много-много картонных коробок, с которыми можно играть. Заповеди ZFC описывают как раз такой картонный мирок. Не исключено, что Цермело был домашним котом Френкеля, и они писали статьи вдвоем. Ведь в математике никто не знает, что ты кот. Пуанкаре, вон, прямо в аудитории молоко из блюдечка лакал - а никто так ни разу и не догадался.
Аноним 14/10/20 Срд 14:01:05 74723 122
>>74716

Подумай про мотивацию, нахуй они нужны все вместе и каждая из них. В википедии подробные исчерпывающие комментарии к каждой аксиоме.
Аноним 08/11/20 Вск 13:07:30 75734 123
Смешно и грустно. Опчик, будучи частью системы, даже не осознает, что озвученный им (на самом деле не им, но это не суть важно) идеал является неотъемлемой частью того самого пиздеца, с которым якобы призван бороться. Пиздец всеобъемлющ и не ограничен одним только получением корок. Пиздец - это тифарет, мехмат, цитаты арнольда, ежедневный случайный еврей, серия пять колец, листочки, нму, садовничий, математика в 57 школе, библиотечка квант, список вербицкого, dxdy, трэш-ролики, каледин, льготы БВИ, не совсем наивный вавилов, ленинградские математические кружки, борьба русофобов с антисемитами, обязательный прасолов и принудительный шень, летняя школа в дубне, собеседование в шад и прочее, и прочее, и прочее - тошное, душное, мрачное и ветхое, как Талмуд или стенографический отчет XXVII съезда Коммунистической Партии Советского Союза. Завещанные правнукам бесконечно устаревшие войны прадедов. Все это должно было исчезнуть давным давно - но в России в очередной раз наступили застойные времена, а застойным временам присущи застойные явления. И эпоха опять занемела, как отсиженная нога.
Аноним 08/11/20 Вск 13:31:18 75735 124
>>75734
>Пиздец - это
в перечисленном действительно немало плохого, но много и хорошего
поскольку внятая аргументация отсутствует, похож он только на шизоидный бред (если только автор не ненавидит математику в принципе)
Аноним 08/11/20 Вск 14:59:42 75736 125
>>75735
У тебя с аргументацией тоже не ахти. Полагаю, что если копнуть ее поглубже, она целиком сведется к синдрому утенка.

В любом застое можно найти много хорошего. По мнению многих, наш дедушка Брежнев хороший был вождь. Но застой все равно остается застоем - и наличие хорошего в нем лишь подчеркивает отсутствие чего-то гораздо более лучшего. Не говоря уже о том, что следующая стадия застоя - гниение и сепсис. Думаю, смысол месседжа примерно в этом.
Аноним 08/11/20 Вск 15:34:23 75737 126
>>75736
отчего же не ахти, поток бессвязных обличений всего подряд без пояснения, что во всём этим не так, я называл похожим на бред, это вполне последовательно

сейчас твоя идея (или не твоя, если ты только апологет) проявляется лучше, мол, это всё застой и никакого нового, потому плохо. ну такое себе
08/11/20 Вск 16:10:43 75738 127
>>75736
>В любом застое можно найти много хорошего.
Тогда список выше - абсолютно нерелевантен, если в застое можно найти всё, что угодно. Нахуй городить какую-то шизу, если по твоим же аргументам всё это не является характеристическим для застоя? Так и скажи прямо, в чём причина, в чём выражается, чем плохо, и как решить. А то это как сказать "пиздец - это гетто, геи, и говно" в дискуссии про "застойные времена" демократических строев. То есть, конечно, связь какую-то найти и можно, прищурившись и сделав пару логических ошибок, но звучит как совершенная нелепица воннаби хвилософа.
Аноним 08/11/20 Вск 16:12:41 75739 128
>>75734
А ты сам с какой стороны это пишешь? Как человек, который прошел через это все и разочаровался, или как человек, который пытался влезть в математику и не осилил?
Аноним 08/11/20 Вск 16:15:25 75740 129
>>75739
ты забыл третий вариант, самый вероятный:
>или как не очень умный тролль?
Аноним 08/11/20 Вск 16:17:31 75742 130
>>75735
Это лишь часть пасты из /un. Вот вся:

Отвечу в свойственном ОПу скушном телеграфном стиле: в вузах должны преподавать вузовские преподаватели. Потому что снег идет зимой и не идет летом, Волга впадает в Каспийское море, а лошади кушают овес. Чрезвычайно содержательное суждение, не так ли?

На самом деле, действующих ученых пора бы уже оставить в покое, а не вешать на них неподъемную педагогическую нагрузку. Хотят преподавать - пусть преподают. Но они в большинстве своем не хотят - просто не хотят, а не потому что им кто-то там на кафедре что-то запрещает. А заманивать этих нехочух рублем денег не напасешься.
Кроме того, при разработке учебной программы следует как минимум различать завершенную и актуальную математику, а также математические факты и математические навыки (те самые habits of mind, относительно которых столь много было сказано и до сих пор ничего не сделано). Массив фактов завершенной математики может превосходно изложить даже средней руки аспирант. Для постановки навыков нужно совместное участие как профессиональных математических педагогов, так и действующих ученых, передающих личный опыт в рамках мастер-классов. Актуальной математике место только на старших курсах - и вот ее-то как раз должны преподавать те самые действующие или недавно ушедшие в отставку ученые, прошедшие дополнительные курсы педагогической подготовки.

Все вышеизложенные факты тривиальны и обмусолены со всех сторон еще в 50е-60е годы двадцатого века. Правда, выводы их них были сделаны различные. В СССР сделали ставку на олимпиадную математику (пиздец) и спецшколы с системой Константинова (пиздец, помноженный на восемь, хотя все равно в разы лучше системы физтеха и прочих форм ада). В РФ ситуация дополнительно осложнена тем, что созданную в Союзе систему отбора и подготовки талантливой молодежи прибрали к рукам банки и айти-корпорации, которые выкачивают для личных галерных нужд все потенциально способные делать науку мозги. Сделать с этим пиздецом давно ничего нельзя, его масштаб слишком велик. Даже матфак, при всех его достоинствах, это всего лишь утлая лодчонка с дырявым парусом, терпящая бедствие в море, полном умных, жадных и злых акул. И пока опчик издалека грозит им кулачком и топает ножками, они спокойно и без суеты доедают его менее расторопных ровесников.

Смешно и грустно. Опчик, будучи частью системы, даже не осознает, что озвученный им (на самом деле не им, но это не суть важно) идеал является неотъемлемой частью того самого пиздеца, с которым якобы призван бороться. Пиздец всеобъемлющ и не ограничен одним только получением корок. Пиздец - это тифарет, мехмат, цитаты арнольда, ежедневный случайный еврей, серия пять колец, листочки, нму, садовничий, математик в 57 школе, библиотечка квант, список вербицкого, dxdy, трэш-ролики, каледин, льготы БВИ, не совсем наивный вавилов, ленинградские математические кружки, борьба русофобов с антисемитами, обязательный прасолов и принудительный шень, летняя школа в дубне, собеседование в шад и прочее, и прочее, и прочее - тошное, душное, мрачное и ветхое, как Талмуд или стенографический отчет XXVII съезда Коммунистической Партии Советского Союза. Завещанные правнукам бесконечно устаревшие войны прадедов. Все это должно было исчезнуть давным давно - но в России в очередной раз наступили застойные времена, а застойным временам присущи застойные явления. И эпоха опять занемела, как отсиженная нога.
Аноним 08/11/20 Вск 16:37:13 75744 131
>>75742
где-то я это видел и вне /un (о существовании которого даже не догадывался)

на тифаретнике, наверно
может быть, даже дмитрий павлов написал, как-то похоже даже очень
Аноним 08/11/20 Вск 16:37:55 75745 132
>>75744
не, не павлов, раз "опчик" и маты

это я просто не читая предположение сделал
Аноним 08/11/20 Вск 16:39:17 75746 133
>>75745
>>75737
и конечно как всегда забыл что это копипасты тред
мог бы и догадаться, что там большое что-то накатано
Аноним 08/11/20 Вск 17:19:04 75747 134
>>75745
>опчик
>о пичк
>о пучк
>опчук
Аноним 08/11/20 Вск 17:50:15 75748 135
>>75747
посмеялся, спасибо
Аноним 11/11/20 Срд 09:05:11 75877 136
Решай олимпиадные задачи, говорили они. Это очень полезно, повторяли они.

Нет, это не полезно. Скорее даже вредно. Мозг очень быстро приучается бессознательно узнавать задачи по их внутренней сути и запускать процессы решения в обход понимания. Иногда ситуация доходит до полного абсурда: ты выходишь из состояния потока, смотришь на решение какой-то там по счету задачи и вдруг осознаешь, что понятия не имеешь, о чем именно в задаче спрашивалось. То есть решение задачи уже найдено, а ты даже условия еще не читал.

Олимпиадная математика была ошибкой. Причем системной. Сама концепция полностью неверна. Это что-то на уровне зубрежки перед экзаменом - наукой доказано, что это самый неэффективный метод обучения из всех возможных, но миллионы студентов до сих пор интуитивно считают, что зубрежка это нормально, "ну в памяти ведь в любом случае что-то остается". Точно так же с точки зрения интуиции вроде бы совершенно ясно, что олимпиады это хорошо - "ну мозг ведь развивается".

Только по моим ощущениям мозг не столько развивается, сколько превращается в мешок с блохами. Прыг-прыг-дирихле. Прыг-прыг-непрерывность. Прыг-прыг-инвариант. Прыг-прыг. А потом такой вот мешок с блохами поступает на какой-нибудь фопф и учит всех вокруг жить. Ну ладно фопф, его не жалко, но меня-то за что? Бр-р-р.
Аноним 13/11/20 Птн 23:19:37 75994 137
Коль я здоровый, не убогий, зачем мне группы гомологий?
from dxdy
Аноним 14/11/20 Суб 22:50:43 76033 138
>>75994
>Коль я глупый и убогий, зачем мне группы гомологий?
органичнее бы звучало

"здоровый, не убогий" -- как-то не очень, притянуто и натужно
Аноним 24/11/20 Втр 10:45:33 76396 139
>>76033
http://www.math.nsc.ru/conference/mml/Golub/ONECURC.HTM
На своём втором курсе я внёс свой посильный вклад в топологию, и, похоже, на его фоне все мои последующие достижения в математике, как это иногда бывает, выглядят жалким эпигонством. Уже в зрелом возрасте я несколько раз испытывал сильные приступы головокружения от того, что маститые коллеги, на которых мы по молодости смотрели с трепетом, спрашивали меня: “Это не Вы ли написали:

Коль я здоровый, не убогий,
зачем мне группы гомологий?”
Другие мои топологические результаты таким успехом что-то не пользовались.
Аноним 24/11/20 Втр 10:58:53 76397 140
>>76396
да, интересная страничка
Аноним 30/11/20 Пнд 12:22:57 76660 141
Люди приходят на матан, изучают какие-то непонятные производные, какие-то градиенты; приходят на комплан - поверхности Римана, преобразования Фурье; на алгебре - зачем-то матрицы, обратные матрицы, поля, вычеты, кольца, торы, эллиптические кривые. Все относятся к этому, как к неизбежному злу - выучил и забыл.
А потом начинается веселье. Оказывается, что если хочешь зарабатывать много в Machine Learning, то будь добр знать градиентный бустинг, хочешь программно обрабатывать изображения - понимай дискретные преобразования Фурье, свёртки, вейвлеты, хочешь работать в геймдеве - понимай матричные операции, хочешь работать со звуком - снова свертки и преобразования Фурье, хочешь податься в криптографию - знай теорию групп и всё рядом лежащее.
Аноним 30/11/20 Пнд 14:48:47 76674 142
>>76660
дихлофос несите сюда скорее преобразование Фурье он на поверхности римана изучает, лол
Аноним 01/12/20 Втр 23:24:23 76732 143
>>76660
> Оказывается, что если хочешь зарабатывать много в Machine Learning, то будь добр знать градиентный бустинг, хочешь программно обрабатывать изображения - понимай дискретные преобразования Фурье, свёртки, вейвлеты, хочешь работать в геймдеве - понимай матричные операции, хочешь работать со звуком - снова свертки и преобразования Фурье, хочешь податься в криптографию - знай теорию групп и всё рядом лежащее.
оказывается, что для всех этих дисциплин достаточно знать просто определения соответствующих терминов, а вся низлежащая теория нахер не нужна.
Аноним 06/12/20 Вск 09:07:32 76879 144
Матшкольник, как домашний котик, должен быть заранее приучен к листочкам - иначе шок, стресс, облезлая шерстка и отчисление из котиков.
Аноним 06/12/20 Вск 23:48:12 76906 145
>>76864
>Я погромист и я илита, а те кто работает физически на заводах - те унтерменши.
Типичный менталитет постсовка. В совке было уважение ко всем профессиям, а рашке же ты по умолчанию говно.
Аноним 07/12/20 Пнд 00:39:54 76908 146
hqdefault (1).jpg 18Кб, 480x360
480x360
>>76906
> В совке было уважение ко всем профессиям
Нет. Была номенклатура, и было говно. Сорта у говна действительно были, но уважали только уважаемых людей.
Аноним 07/12/20 Пнд 06:44:23 76916 147
16073010302930.png 116Кб, 663x607
663x607
Аноним 11/12/20 Птн 17:55:25 77141 148
>>76908
Сейчас методы в рашке и форма управления осталась совковой, только говно теперь считает себя и окружающих говном.
Аноним 23/12/20 Срд 23:16:44 78005 149
Сканави в этом плане можно разве что шкаф подпереть. Он годится только для повторения школьной программы. Среднестатистический матшкольник перед поступлением на матфак в течение 2-4 лет осваивает темы, которые в эту программу вообще не входят.
https://math.hse.ru/podgotovka

Матшкольник, как домашний котик, должен быть заранее приучен к листочкам - иначе шок, стресс, облезлая шерстка и отчисление из котиков. Вот, например, сборники листочков прежних лет, в которых покрывается школьная программа, кружково-олимпиадные темы, материал первого курса университета.
Давидович Б.М., Пушкарь П.Е., Чеканов Ю.В, "Математический анализ в 57-й школе. 4-годичный курс".
Голенищева-Кутузова Т.И., и др., "Элементы математики в задачах", Ч.1, 2.
Доценко В. (ред.), "Задачи по математике, предлагавшиеся ученикам 57-й школы".

Алгебра и анализ для матшкольников. Каждый сборник (кроме Алексеева) содержит порядка 1000 задач. Чтобы стать матшкольником на минималках, нужно решить хотя бы 70-80% из них.
Алексеев В.Б., "Теорема Абеля в задачах и решениях".
А.А. Заславский, А.Б. Скопенков, М.Б. Скопенков, "Элементы математики в задачах: через олимпиады и кружки - к профессии" (на либгене висит черновая версия от 2017.04.15).
Алфутова Н.Б., Устинов А.В., "Алгебра и теория чисел для математических школ".
Гашков С.Б., "Современная элементарная алгебра в задачах и упражнениях".
Прасолов В.В., "Задачи по алгебре, арифметике и анализу".

Классическая кружковая литература. Олимпиадный фольклор, баяны, штучки, дрючки, фокусы и покусы без которых никуда (Правда никуда, проверено. Примерно 30% задач первого курса берутся из фольклора и содержат какой-нибудь фокус-покус, не зная которого ты их фиг решишь).
Серия "Школьные математические кружки".
Генкин С.А., Итенберг И.В., Фомин Д.В., "Ленинградские математические кружки. Пособие для внеклассной работы".

Плюс нужно знать азы теории множеств - ибо это язык, на котором будет идти изложение предмета на начальных курсах.
Виленкин (совсем для детишек).
Шень, "Начала теории множеств".
Вавилов, "Не совсем наивная теория множеств" (осторожно, духовность).
Аноним 24/12/20 Чтв 11:40:12 78038 150
урс шрайбер.png 248Кб, 1191x954
1191x954
урс шрайбер2.png 179Кб, 1521x953
1521x953
Аноним 24/12/20 Чтв 14:52:55 78045 151
>>78038
Ох уж категорные пыни даже тифарет достали.
Аноним 24/12/20 Чтв 14:55:06 78046 152
>>78045
Претензия не к категорности, а к некошерности. У Urs Schreiber категории выбегаллы.
Аноним 24/12/20 Чтв 16:06:40 78047 153
>>78046
> категории выбегаллы
Что блядь?
Аноним 24/12/20 Чтв 16:11:06 78048 154
>>78047
Бумерский мем.
Аноним 24/12/20 Чтв 16:35:28 78049 155
>>78047
— Эй, девка… эта… молока давай! Лей, значить, прямо сюда, в отрубя… Силь ву пле, значить…
Стелла торопливо подхватила ведро и плеснула в кювету обрат.
— Эх! — воскликнул профессор Выбегалло. — Посуда мала, значить! Ты, девка, как тебя, эта, прямо в чан лей, будем, значить, из чана кушать.
Стелла стала опрокидывать ведра в чан с отрубями, а профессор ухвативши кювету, как ложку, принялся черпать отруби и отправлять в пасть, раскрывшуюся вдруг невероятно широко.

https://ru.wikipedia.org/wiki/Выбегалло
Аноним 24/12/20 Чтв 17:00:24 78054 156
100% народа, спорящего в интернетах про теоремы Гёделя, не в состоянии открыть учебник и прочитать хотя бы формальную формулировку теорем.

те же, кто хотя бы прочел и понял, про что теорема, уже ни про что спорить не будут, поэтому они в спорах не участвуют, и вы их, соответственно, не наблюдаете.

вышенаписанное следует считать "последней метатеоремой имени Гёделя".
Аноним 06/01/21 Срд 21:37:04 78693 157
IMG202101062133[...].jpg 128Кб, 1080x785
1080x785
Аноним 06/01/21 Срд 22:47:46 78696 158
>>78693
мне как-то трудно в это поверить, будто бы пенской позабыл о приличии

ремарка "математик не то чтобы замечательный, но хороший" также наводит на мысли насчёт реальной объективности отзыва
Аноним 06/01/21 Срд 22:57:05 78697 159
>>78696
почитал цитаты вроде бы его по ссылке, действительно, ничего подобного я от него не слышал
Аноним 07/01/21 Чтв 03:45:27 78698 160
>>78697
К сожалению, информация подтверждается.
Аноним 07/01/21 Чтв 10:39:37 78701 161
>>78693
Между прочим, был живым свидетелем аналогичного поведения одного алгебраиста в СПб. Только там модальность переключалась между студентами матмеха и погромистами. Мата в таком количестве хоть и не было, но общий тон отношения был совершенно отвратительным.

Личность это все-таки маска, спектр допустимого поведения - а не какая-то там подлинная сущность. Двачерам - то есть людям, которые в одном треде кидают друг в друга говном, а в соседнем целуются в десны, - это должно быть прекрасно известно.

Так что чувак просто тред попутал.
Аноним 07/01/21 Чтв 20:38:43 78704 162
>>78698
Ужасно, да
упало теперь моё мнение о нём
Аноним 11/01/21 Пнд 16:06:03 78827 163
>>78701
Не преподает ли он сейчас на факультете стасаикн?
Аноним 14/01/21 Чтв 03:59:18 78990 164
С эмблемы мехмата МГУ исчезнет лента Мёбиуса

Руководство старейшего математического факультета страны решило сменить имидж. Причиной такого решения с «Панорамой» поделился декан А.И.Шафаревич:
"После утомительных совещаний и обсуждений мы пришли к выводу, что 60-е (так называемый «золотой век мехмата») вернуть не получится, с тех пор внешний облик факультета изменился навсегда. Конечно, аналитическая школа на мехмате по-прежнему сильна, но ярких достижений мирового уровня в геометрии, топологии, алгебре сейчас на нашем факультете всё меньше и меньше. Догонять передний край науки — подвиг, на который способен отдельный заинтересованный человек, но уж точно не такой огромный факультет, как наш. Так что, оставляя в логотипе только знак интеграла и таблицу 7×7, мы ещё раз подчеркиваем: в перспективном плане развития факультета ставка делается на анализ и на прикладную (но высокоуровневую и наукоёмкую!) математику.

Заодно убьём двух зайцев: не придётся сплавлять любителей абстрактной зауми без приложений на кафедры-мясорубки со словами "высшая" в названии (чтобы не обижать коллег, обойдусь без точных наименований), если они сами испугаются чуждой им идеологии и побегут поступать на известный факультет, где кучкуются такие же любители современного искусства"
Аноним 14/01/21 Чтв 04:17:35 78991 165
Аноним 14/01/21 Чтв 06:16:54 78992 166
>>78990
>в перспективном плане развития факультета ставка делается на анализ и на прикладную (но высокоуровневую и наукоёмкую!) математику.

ну что поделать
возможно, у них нет таких зарплат, как в вышке (известно, что в ней платят больше), потому они намерены ориентироваться на области, где можно больше заработать

>>78991
немало интересного, как по мне. выбор есть
Аноним 14/01/21 Чтв 10:02:14 78994 167
Минский называл эту способность умением учиться учиться. Ее можно отнести к тем навыкам, которые сейчас называют coping strategies - то есть приемами борьбы с трудностями. Что касается математики, то показано, что трудности с математикой возникают у ВСЕХ студентов, но успешные студенты отличаются от всех остальных наличием индивидуальных методов борьбы с трудностями - они умеют диагностировать свои косяки и выстраивать хорошо тюнингованные пайплайны обучения.

Проблема с трансфером этих методов заключается в том, что методы успешных студентов настолько идиосинкратичны (они слишком СТРАННЫЕ), что другому человеку очень сложно их принять чисто психологически. Например, кто-то говорит, что "убрал из комнаты все, кроме стола и учебника, чтобы учебник стал единственным интересным объектом в комнате". Кто-то "привинтил к стене металлический лист и покрыл стикерами с лекционными материалами". Кто-то "в конце каждой главы рисовал на ватмане схему всего изученного и потом несколько десятков раз проходил ее в разных направлениях". Кто-то "с интервалом в неделю прорешивает одни и те же задачи". Кто-то "представляет себе объект, как кристалл из конструктора и последовательно применяет к нему все трансформации из заранее составленного списка". И т. д. Во-первых, со стороны это выглядит как полная шиза, а во-вторых, носители этой "шизы" прекрасно понимаю, что другой реакции ждать не приходится - поэтому подобные странные методы и идеи обычно остаются неизвестными окружающим.

Есть и еще один аспект - тоже скрытый от внешних глаз, но скрытый немного в другом смысле (кстати, Минский его тоже упоминает). Нам, как внешним наблюдателям, очень легко понять, что человек делает для достижения своих целей - но в миллион раз труднее понять, что он НЕ делает для достижения этих же целей. Например, мы смотрим на очень успешного однокурсника и видим, что он как заведенный ходит на все лекции и семинары. Мы тоже начинаем как заведенные ходить на все лекции и семинары - только проходит месяц за месяцем, а толку почему-то никакого. И только очень пристальный анализ может показать, что наш успешный поц посещает лекции только потому, что учеба на дому лично для него оказалась неэффективной - и он ходит на лекции, чтобы НЕ учиться дома. В то время как для нас самостоятельные занятия могут, напротив, оказаться наиболее эффективной стратегией - и тратя время на очное посещение лекций, мы попусту растрачиваем массу драгоценного времени. И это только самый простой пример - что касается приемов обучения, решений задач и исследовательской работы, то там могут быть тысячи нюансов и тысячи вещей, которых люди НЕ делают, чтобы быть успешными.

И подобная ситуация характерна не только для математики - она в целом является характерной, когда дело касается аномальных человеческих достижений. Можно взять в качестве примера полиглотов. Про них известно несколько вещей. Многие полиглоты в школе оказываются вообще ни бум-бум, а начинают изучать языки только будучи сильно взрослыми - то есть дело не в образовании и не во врожденных способностях. Если полиглот знает языки, то он знает сразу МНОГО языков - то есть он с вероятностью 100% разработал некоторый пайплайн. В среднем +1 язык приобретается за два года неторопливой работы (если поторопиться, то можно выйти на бытовой разговорный уровень где-то за 14 недель). Каждый полиглот имеет свой персональный метод, который ОЧЕНЬ сильно отличается от всего, что можно встретить в официальной системе образования. Большинство этих методов со стороны выглядит как шиза ебаная и встречает массу препятствий при попытке трансфера - люди либо поднимают человека на смех, либо упрекают в инфоцыганстве, либо крутят пальцем у виска.

Смысол этой пасты в том, что чужие методы тебе недоступны (о них упоминают очень редко и только вскользь, они выглядят нелепо и странно и их очень трудно усвоить), официальную методологическую литературу ты устанешь читать, а на разработку персональной методики у тебя, судя по всему, не хватит ни дерзости, ни времени, ни желания. Ты до конца будешь разрываться между тупыми, но успешными дегенератами из тиктока, и гениальными, но опять же успешными, гениями с матфака и фкн - только ни к одному из этих лагерей так и не пристанешь, лишь с головой погрязнешь в вечном баттхерте. Так что терпи @ страдай - либо будь дерзновен.
Аноним 14/01/21 Чтв 20:43:32 79014 168
Аксиома подстановки это эзотеpизм!

Аксиома говоpит вот что. Пусть ф - пpавильно постpоенная фоpмула языка теоpии множеств пеpвого поpядка, в котоpую буква B не вxодит свободно. Пусть p - её стpочка паpаметpов. Tогда для любого множества A и любого набоpа паpаметpов p если ф функциональна относительно A, то существует множество B, состоящее в точности из такиx элементов, котоpые являются ф-обpазами элементов A.

Эта аксиома является эзотеpической потому, что позволяет легко доказывать существование множеств, о пустоте или непустоте котоpыx в пpинципе ничего сказать нельзя.

Hапpимеp, пpавильно постpоенной фоpмулой является фоpмула "мощность x стpого больше алеф-0, но стpого меньше континуума". Пpименив эту фоpмулу к какому-нибудь каpдиналу большему, чем континуум, мы получим множество каpдиналов M, пустота котоpого pавносильна континуум-гипотезе. Множество M существует, но его элементы никакими сpедствами не могут быть изучены. Hелепость какая-то.

А ведь ф - любая пpавильно постpоенная фоpмула. Котоpая к тому же может зависеть от любого числа паpаметpов. Это, если вдуматься, позволяет буквально доказать существование Ктулxу.

В самом деле, доказательство сводится к точному теоpетико-множественному описанию того, что мы понимаем под Ктулxу. Если C(x) - утвеpждение, что множество x является Ктулxу, то может быть постpоена фоpмула f(x, y), котоpая функциональна по x и каждому иксу сопоставляет Ктулxу. Пpименив эту фоpмулу к какому-нибудь одноэлементному множеству, мы докажем, что существует множество, единственным элементом котоpого является Ктулxу.

Более того, если считать, что мы имеем пpаво пеpенумеpовать Великиx Дpевниx оpдиналами, то мы можем доказать существование любого напеpед заданного количества экземпляpов Ктулxу. Даже алефа с тpансфинитным нумеpом pазличныx ктулx.
Аноним 14/01/21 Чтв 23:50:44 79025 169
>>79014
Исходная паста написана на правах дискуссионного лытдыбра и не является мнением, которое автор готов защищать на академическом собрании.
Аноним 15/01/21 Птн 13:56:14 79038 170
>>78990
>>78992
>с «Панорамой» поделился
Это комедийные фейкньюс, на всякий случай.
Аноним 15/01/21 Птн 15:53:33 79057 171
>>79038
понятно
на дваче куда не вступишь попадёшь в говно
Аноним 15/01/21 Птн 22:30:14 79075 172
>>79057
Зря ты так. Смешная шутка же.
Аноним 16/01/21 Суб 04:02:33 79089 173
>>79075
слишком уж в ней много правды
Аноним 16/01/21 Суб 06:52:11 79090 174
>>78990
Причём писал человек в теме, судя по всему.
Аноним 23/01/21 Суб 09:32:18 79424 175
основания гомот[...].png 42Кб, 657x208
657x208
Аноним 23/01/21 Суб 12:51:46 79427 176
>>79424
дай ссылку на жж
Аноним 23/01/21 Суб 20:04:51 79460 177
Листочки это самый эффективный педагогический инструмент, когда-либо созданный человеческим умом.

Арнольд учил своих асов-аспов по листкам (конкретное тому подтверждение - Тривиум).

Перельман учился по листкам Рукшина.

Основатель сибирской школы алгебры и логики – Мальцев – учился по листкам.

Шафаревич учился по листкам. В итоге поступил сразу же на последний курс МГУ.

Вся знаменитая группа математиков золотого века Мехмата училась по листкам (см. семинары Лузина).

Сам Лузин учился по листкам (у студента-репетитора).

Софья Ковалевская училась по листкам Вейерштрасса. Впоследствии она активно решала проблемы, поставленные Лондонским матобществом (то тоже были листки).

Чебышёв учился оперировать с многочленами по листкам.

Эндрю Уайлс учился по листкам. В его листке была только одна задача – теорема Ферма (по его словам, он впервые познакомился с теоремой в библиотеке, а затем выписал утверждение к себе в листок).

Гротендик, Лебег, Пикар, Адамар, Вейль – все они учились по листкам.

Все бурбаки коллективно решали листки.

Младшие Бернулли учились по листкам старшего Бернулли. Самый старший Бернулли купил первые математические листки у голландского торговца.

Физики тоже: все ученики Ландау учились по листкам Ландау (см. теорминимум Ландау).

Фейнман учился по листкам в калтехе.

Дирак, Гейзенберг, Борн, Шредингер – все они решали много листков в своих университетах.

У Фарадея был листок, на котором было написано: «объединить электричество с магнетизмом». Он решал много листков из частных и публичных библиотек (он был библиотекарем).

Галилей, Ньютон, Лейбниц – все они учились по листкам, составленными физиками и различного рода арабами, жившими до них.

Аристотель учил Александра Македонского по листкам ВШЭ (!). Из-за этого полководец стал слишком много внимания уделять математике, пренебрегая такими полезными предметами, как география, биология, философия (это было подмечено Арнольдом).

Да и вообще, последними словами Архимеда были: «Не трогай моих листков», что было искаженно переведено современными лингвистами, как «Не трогай моих чертежей».

Евклид учился по листкам, составленным египетскими жрецами.

А вот откуда египетские жрецы нашли первые листки – этого никто не знает. Эту величайшую тайну безуспешно пытаются открыть египтологи. Хотя, после открытий Шампольона, возникло предположение, что египтяне переняли систему листков у древних шумеров.

А вот в мехмате МГУ, как я слышал, по листкам не учат…
Аноним 24/01/21 Вск 06:12:08 79472 178
>>79460
В голос.
>Самый старший Бернулли купил первые математические листки у голландского торговца.
>Не трогай моих листков»
Аноним 24/01/21 Вск 08:49:47 79473 179
>>78693
Классика для пидорах. Гельфанд тоже, в союзе матом направо и налево кидался, в США же всем очки облизывал.
Аноним 24/01/21 Вск 12:40:53 79476 180
>>79424
переписать основания гомотопий?
где-то это уже было... ах да, Воеводский.
Аноним 24/01/21 Вск 14:55:40 79478 181
>>79476
Воеводский вообще все основания переписать хотел
Что здесь имеет в виду Каледин, не очень понятно, но да, настораживает
Аноним 24/01/21 Вск 15:55:45 79480 182
Снимок экрана 2[...].png 129Кб, 488x716
488x716
Аноним 24/01/21 Вск 18:56:45 79482 183
>>79480
Вербит что таракан?
Аноним 24/01/21 Вск 20:40:47 79483 184
>>79482

Вот фон Нейман таракан или нет? А Гельфанд? Программирование возникло из математики, все хорошие программисты это бывшие математики.
Аноним 25/01/21 Пнд 00:01:49 79492 185
>>79483
На каком языке математики прогают? На с++?
Аноним 25/01/21 Пнд 06:46:53 79496 186
>>79492
По работе? Чистые ни на каком, прикладные зависит от области. Например, фортран в физике и квантовой химии, R или Stata в эконометрике и статистике, и т.д.
Для собственной пользы? Опять же зависит от целей, питон хорош для быстрых скриптов, например.
Вопрос бессмысленен, вобщем.
Аноним 25/01/21 Пнд 07:43:55 79500 187
>>79492
Не все математики умеют прогать, только некоторые. Ещё некоторые думают о себе, что умеют, но на самом деле не умеют. inb4 Хаскель: вообще-то у фундаментальных исследователей он непопулярен, слишком кринжевый
Аноним 25/01/21 Пнд 10:13:31 79502 188
Даже не все исследователи CS умеют прогать
Аноним 25/01/21 Пнд 12:39:28 79504 189
>>79502
даже не все программисты умеют прогать
Аноним 25/01/21 Пнд 12:42:32 79506 190
>>79478
>Воеводский вообще все основания переписать хотел
хотел все, но назвал их почему-то "Homotopy TT", что как бы намекает

>Что здесь имеет в виду Каледин, не очень понятно, но да, настораживает
Каледин написал это как раз после смерти Воеводского. совпадение?
Аноним 25/01/21 Пнд 13:37:15 79511 191
Сдвиг в сознании это хорошо. Живое должно расти, а не костенеть. Для закрепления сдвига можно сходить на википедию и убедиться, что методов обучения и в самом деле существует >1.
https://en.wikipedia.org/wiki/Category:Learning_methods

Кое-что можно порекомендовать более конкретно. Сходи по ссылке:
https://en.wikipedia.org/wiki/Active_learning
И посмотри главу The principles of learning. Там будут указаны ссылки на статьи, с которыми имеет смысл ознакомиться (скорее всего, все есть на сайхабе).

Эффективные методы кажутся большинству людей контринтуитивными и неестественными - но это одно из наших когнитивных искажений, и его придется сознательно преодолевать. Презентация деда №1 о контринтуитивности эффективных методик.
https://www.youtube.com/watch?v=oxZzoVp5jmI[РАСКРЫТЬ]

Классический курс LHTL, для общего понимания происходящего.
https://ru.coursera.org/learn/learning-how-to-learn
https://libgen.is/book/index.php?md5=7E96F5A45268F2794BDA804E23616DF5

Несколько полезных статей из вики Возняка. Хотя дед №2 давно поехал на интервальных повторениях, но у него там до сих пор много годноты можно найти.
http://www.super-memory.com/articles/20rules.htm
http://www.super-memory.com/articles/theory.htm
https://supermemo.guru/wiki/Science_of_sleep

Более конкретно по изучению математики. Все книги выглядят как сборник капитанских советов, но ничего лучше для старшеклассников и студентов пока не нашел. (Хотя их восприятие как "очевидных" тоже является когнитивным искажением. Не так уж они очевидны, на самом деле.)
Kevin Houston, "How to think like a mathematician".
Lara Alcock, "How to Study as a Mathematics Major".
J. Mason, L. Burton, K. Stacey, "Thinking Mathematically".
Кринжовая методичка чисто по техническим инструментам.
Bindner D., Erickson M., "A student's guide to the study, practice, and tools of modern mathematics".
Можешь еще посмотреть серию книг от Steven G. Krantz по изучению математики, ее преподаванию и проч. Он много чего понаписал, некоторым нравится.

Довольно известные в узких кругах книги, ориентированные на методологию.
Акцент на примерах и их самостоятельной генерации.
Anne Watson, John Mason, "Mathematics As A Constructive Activity: Learners Generating Examples".
Акцент на самостоятельной генерации проблем.
Stephen I. Brown, Marion I. Walter, "The art of problem posing".

Алсо, по поводу изучения языков - на тот случай, что если кто-то до сих пор думает, что это так дохуя сложно.
https://www.youtube.com/watch?v=g-1oHIneSNw&ab_channel=PolyglotConference
https://www.youtube.com/watch?v=illApgaLgGA&ab_channel=Poly-glot-a-lot
Аноним 25/01/21 Пнд 15:20:54 79514 192
>>79482
>>79483
>>79492
>>79500
Математика это язык программирования с более развитой типизацией.
Аноним 25/01/21 Пнд 17:39:30 79516 193
>>79514
С более хуевой документацией.
Аноним 25/01/21 Пнд 20:17:02 79519 194
АААААААААААААААААААААААААААААА АААААААААААААААААА
ТАААААААААРААААААКААААААААНННННННЫЫЫЫЫЫЫЫЫЫЫ
Аноним 26/01/21 Втр 10:44:41 79545 195
>>79506
>но назвал их почему-то "Homotopy TT", что как бы намекает
намекает на то, что он хочет использовать термины/идеи из теории гомотопий

>Каледин написал это как раз после смерти Воеводского. совпадение?
несколько лет прошло?
Аноним 26/01/21 Втр 11:05:35 79546 196
>Сегодня, в XXI веке, чтобы избежать неминуемого лошизма, люди пишут "Доброе время суток!". Получатель письма сам выглянет в окно и подставит себе нужное значение, типа.

Какая есть математическая аналогия у этого явления? Наверняка есть какой то морфизм
Аноним 26/01/21 Втр 17:35:58 79572 197
>>79546

Конечно есть, называется "целочисленное деление на 6".
Аноним 03/02/21 Срд 05:24:50 79879 198
Расширение модулей переломлено пополам
А наш батюшка Арнольд совсем усох
Он разложился на прямые слагаемые и на пересечение неприводимых подмодулей
А надстройка все идет и идет по конусу
И всё многообразие вложилось в RN
И весь мир состоит из модулей!
Модули над кольцом!
А моя судьба захотела на покой
Я обещал ей не участвовать в гомотопическом хаосе
Но на фуражке на моей коммутативная диаграмма и копредел
Как это трогательно коммутативная диаграмма и копредел
Лихая петля порождает фундаментальную группу
И весь мир состоит из модулей!
Модули над кольцом!
А моей статьёй накормили толпу
Анонимный рецензент нашёл в ней ошибку
Научное сообщество растоптало её на конференции
Так исправляйте же её сами
Потому что весь мир состоит из модулей!
Модули над кольцом!
Один лишь дедушка Шурик хороший был вождь
А все другие остальные такое дерьмо
А все другие враги и такие дураки
Над родною над математикой снег невежества шел
Я купил журнал LJ.ROSSIA там тоже хорошо
Там товарищ Миша там тоже что у нас
Я уверен, что у них то же самое
Весь мир состоит из модулей!
Модули над кольцом!
А при унивалентных основаниях все будет заебись
Их закончат скоро надо только подождать
Там все будет автоматически, там все будет в кайф
Там наверное вообще не надо будет ничего доказывать!
Я проснулся среди ночи и понял, что...
ПУЧОК МОДУЛЕЙ НАД ПУЧКОМ КОЛЕЦ
ПУЧОК МОДУЛЕЙ НАД ПУЧКОМ КОЛЕЦ
ПУЧОК МОДУЛЕЙ НАД ПУЧКОМ КОЛЕЦ
Аноним 03/02/21 Срд 11:42:59 79882 199
>>79879
Говорилкой озвучить и на минусовку наложить надо.
Аноним 24/02/21 Срд 09:16:31 80651 200
Это теоретически невозможно без кватернионов
Если ты используешь матрицы вращения, то ты параметризуешь их чем-то вроде углов Эйлера, то есть 3-тором. Но у группы вращения накрывающее пространство это 3-сфера S3≅SU(2). Кватернионы как раз таки и дают тебе это накрытие (двойное, правда, но это похуй). То есть у параметризации углами Эйлера есть фундаментальный изъян - они не могут правильно описать топологию группы вращений.

Нетривиальная геометрия группы вращений (SO(3)≅RP3) это, кстати, причина многих "неинтуитивных" конструкций в физике, спины там всякие из-за π1(RP3)≅Z2.

Хороший пример того, как программистам никакой интересной математики не обучают. Геймдев форумы завалены вопросами про гимбал лок.

Ах да, забыл всё к гамалогиям свести, мы ж в /math/
Гимбал лок это завуалированный факт о том, что у 3-сферы первые и вторые гомологии тривиальные, а у 3-тора - нет.
Аноним 28/05/21 Птн 13:57:58 83904 201
Твои бурбаки говно и пидоры и источник всего говна и всех пидоров. Если ты идешь по коридору слышишь доклад про мотивы, знай: это фанат бурбаков. Если тебя затрахал и заебал гнетущий совковый бред, инварианты Громова-Виттена, волл-кроссинг и миррор-симметри - будь благодарен бурбакам, вырастившим на навозе в Монпелье всю эту братию, поэтому иди и отсоси у них чтобы по щекам текло. Если наконец на матоверфлоу ты встречаешь абсолютно овцеголового и от рождения закостенелого юзера и фаната нкатлаба, будь уверен: это ебаное бурбакистское быдло, до которого не дотянулись арнольдисты. Поэтому ты мудак, мудак и белая обезьяна и со своими сраными бурбаками не слезешь с березы НИКОГДА. Ведь это пиздец, пиздец, пиздец, в двадцать первом веке мерять мир бурбакистскою мерою и носить в жопе бурбакистские мозги из отрубей и даже мысли не помыслить, что это говно и что говно должно быть уничтожено! Вся мировая история прошла мимо тебя напрасно, сраный дебил!
Аноним 29/05/21 Суб 01:05:14 83930 202
PortraitBourbaki.jpg 124Кб, 750x1057
750x1057
В книжках Бурбаки используется тау-символ. Это разновидность эпсилон-оператора Гильберта. Тау-символ нужен в качестве имени, символизирующего десигнат. Десигнат имени - это идеальный умозрительный объект, способный называться именем.

Например, знакосочетание "τx : x - столица России" способно быть именем для Москвы. Знакосочетание "τx : x - дерево" способно быть именем для берёзы, баобаба, тополя и т.п. Здесь у знакосочетаний есть не только десигнат (умозрительный образ), но и денотат - именующаяся этим именем вещь из объективной реальности, которую можно потрогать. Можно показать какую-нибудь конкретную берёзу, существующую независимо от мыслителя, и сказать: "Вот, это берёза".

Денотат не обязателен. Например, знакосочетание "τx : x - невидимый розовый единорог" имеет десигнат, но не имеет денотата; можно помыслить невидимого розового единорога, но нельзя показать что-то, способное называться этим именем. По крайней мере, так же безоговорочно, как берёза.

Тау-символ не категоричен: связанное им знакосочетание может быть именем для многих разных объектов. Нельзя сказать, что только одна из берёз вселенной является конкретной реализацией идеи "берёза". "τx : x - берёза" обозначает не какую-то одну конкретную берёзу, но может обозначать любую из берёз.

Бурбаки настаивает, что в его формальной теории десигнат полностью выражается как чисто синтаксическое отношение букв между собой; знакосочетания могут иметь денотат, но не обязаны его иметь. Бурбаки не интересуется, есть ли денотат у знакосочетаний, которыми он пользуется.
Аноним 29/05/21 Суб 11:43:35 83938 203
>>19669 (OP)
Дайте ссылку на книги Арнольда. Новичок в математике.
Аноним 29/05/21 Суб 13:04:56 83943 204
>>83938
Книги Арнольда не для новичков.
Аноним 29/05/21 Суб 15:46:28 83949 205
>>83938
Из более-менее воодного есть кватернионы (не помню название), теория катастроф, и абель в задачах (алексеев)
Основные учебники- дифуры и матметоды механики - действительно не для новичков, хотя полистать можно
Аноним 29/05/21 Суб 19:41:33 83955 206
Аноним 31/05/21 Пнд 14:01:57 83991 207
реквестируется паста из како-го то жж, где один кун говорил что математика катится к предсказаниям и мир погружается в мир иллюзий
Аноним 31/05/21 Пнд 14:20:34 83992 208
>>83991
* пакет в пакете
Аноним 09/06/21 Срд 13:01:32 84356 209
Как матрицы - таблицы чисел, так и тензоры - составной объект, образованный из примитивных.

У чисел есть правило раскрытия скобок, (a+b)c = ac+bc.

В векторных пространствах векторы можно только складывать и умножать на число, а перемножать нельзя. Тензорное произведение ⊗ двух пространств - это способ ввести умножение векторов так, чтобы действовало правило раскрытия скобок.

Берёшь базис e1, e2, e3 первого пространства. Берёшь базис w1, w2, w3 второго пространства. Образуешь девять пар объектов, девять формальных символов, девять букв:

(e1, w1), (e1, w2), (e1, w3)
(e2, w1), (e2, w2), (e2, w3)
(e3, w1), (e3, w2), (e3, w3)

Потом переписываешь их с помощью значка ⊗:
e1⊗w1, e1⊗w2, e1⊗w3,
e2⊗w1, e2⊗w2, e2⊗w3,
e3⊗w1, e3⊗w2, e3⊗w3.

Говоришь, что сложение этих пар будет линейно.
(e1+e2)⊗w1 = e1⊗w1 + e2⊗w1
(e1+e3)⊗w1 = e1⊗w1 + e2⊗w3
и т.д.

А потом на эти девять символов натягиваешь векторное пространство. Получится девятимерное пространство с дополнительной операцией ⊗.
Поскольку ты определяешь это сложение самостоятельно и как хочешь, ты действительно можешь это сделать.

Любой вектор v из первого пространства раскладывается по базису первого пространства.
v = ae1 + be2 + ce3.
Любой вектор u из второго пространства раскладывается по базису второго пространства.
u = xw1 + yw2 + zw3.

v можно умножить на u тензорно. Получится
v⊗u = (ae1 + be2 + ce3)⊗(xw1 + yw2 + zw3) =
= ae1 ⊗(xw1 + yw2 + zw3) + be2 ⊗(xw1 + yw2 + zw3) + ce3 ⊗(xw1 + yw2 + zw3) =
= ae1⊗xw1 + ae1⊗yw2 + ae1⊗zw3 + be2⊗xw1 + be2⊗yw2 + be2⊗zw3 + ce3⊗xw1 + ce3⊗yw2 + ce3⊗zw3

Координаты этого девятимерного вектора в построенном пространстве будут
ax bx cx
ay by cy
az bz cz

Если поменять базисы в исходных пространствах, то поменяются девять символов, поэтому поменяются и координаты v⊗u. Но они поменяются понятным образом - с помощью домножения на матрицы перехода.

Зачем это надо? Ну, в приложениях часто используется замена формального значка ⊗ на какую-нибудь осмысленную операцию, которую можно применить к паре. Например, очень часто умножается векторное пространство на пространство линейных функций на этом пространстве. Пары будут иметь вид v⊗f, где v - вектор, f - функция. Тогда операция ⊗ после всех манипуляций заменяется на применение функции к вектору, v⊗f = f(v). Возможны и другие способы заменить ⊗. Тензорное произведение - манипуляции парами символов - упрощает промежуточные выкладки.
Аноним 09/06/21 Срд 13:15:25 84357 210
>>84356
>это способ ввести умножение векторов так, чтобы действовало правило раскрытия скобок.

бред же
тензорное произведение это способ превращать билинейные отображения в линейные, которые намного лучше. по-моему, так
Аноним 09/06/21 Срд 15:37:04 84360 211
>>84356
>(e1+e3)⊗w1 = e1⊗w1 + e2⊗w3
Тут опечатка, должно быть
(e1+e3)⊗w1 = e1⊗w1 + e3⊗w1

>>84357
Тензорно умножать можно не только векторные пространства, но и, например, графы - ситуация, когда ни о какой билинейности речи уже нет. Настоящий смысл тензорного произведения в том, что это просто формальная синтаксическая операция.
Аноним 09/06/21 Срд 15:41:32 84361 212
>>84360
>, но и, например, графы - ситуация, когда ни о какой билинейности речи уже нет.
Во-первых, графы никого не ебут
Во-вторых, тензорное произведение графов есть просто прямое произведение матриц смежности которое, конечно же, линейно (и билинейно)
Аноним 09/06/21 Срд 15:47:14 84362 213
>>84361
> прямое произведение матриц
А эта операция, в свою очередь, дословно совпадает с описанным в >>84356 с точностью до упоминания базиса.
Аноним 09/06/21 Срд 16:08:14 84363 214
>>84362
Прямое произведение матриц - это не
> просто формальная синтаксическая операция.
и тут
> графы - ситуация, когда ни о какой билинейности речи уже нет
ты был не прав с точки зрения математики
так что ты не прав, уж признай это и порадуйся тому, что ты узнал что-то новое о своих графах

я, кстати, другой анон
Аноним 09/06/21 Срд 16:11:50 84364 215
>>84363
Наверное, тебе не стоит говорить от лица математики. Это несколько чересчур дерзко. Мне есть что ответить, но предлагаю перенести дискуссию в другой тред.
Аноним 09/06/21 Срд 17:17:40 84367 216
>>84360
>Настоящий смысл тензорного произведения в том, что это просто формальная синтаксическая операция.
узнаю школу бурбаков, кек. Вся математика вообще набор формальный синтаксических операций и эвристик по их подстановкам.
Аноним 09/06/21 Срд 17:26:47 84370 217
>>84360
>Настоящий смысл тензорного произведения в том, что это просто
абстрактная чепуха без тензорного поля на многообразии.
Аноним 09/06/21 Срд 17:55:21 84373 218
>>84367
Ну да. А производная - это штрих. Иногда связанный с некоторой дополнительной интуицией, но не обязательно. Не считаю, что нужно рассказывать интуицию вперёд синтаксиса.
Аноним 09/06/21 Срд 19:45:27 84383 219
>>84373
>Не считаю, что нужно рассказывать интуицию вперёд синтаксиса.
Адепты китайской комнаты пытаются подражать своим первокультурным героям забывая, какую роль интуиция играла в развитии условных Серра и Дьедонне
Аноним 09/06/21 Срд 20:08:31 84385 220
>>84373
>Не считаю, что нужно рассказывать интуицию вперёд синтаксиса.
Синтаксис без интуиции не имеет никакого смысла.
Аноним 10/06/21 Чтв 12:09:47 84397 221
Синтаксис надо сводить к программированию, где даже парадокс Рассела невозможен, а значит и морочить им голову не нужно.
Аноним 10/06/21 Чтв 20:34:38 84418 222
>>84397

Ну так они и в ZFC невозможен.
Аноним 19/06/21 Суб 20:18:03 84659 223
>>84397
>>84418
если различать язык и метаязык, то не будет самореференции и не будет никаких парадоксов
Аноним 21/06/21 Пнд 15:00:14 84686 224
https://www.nsu.ru/n/mathematics-mechanics-department/documents/courses/
Вот, например, там выложена программа по так называемой "высшей алгебре". Я коснусь только двух моментов из неё.

Во-первых, весь рассказ о кольцах ограничивается двумя словами о евклидовых кольцах. Не о факториальных даже, не о кольцах главных идеалов, а именно о евклидовых. Вся содержательная теория укладывается в два-три определения: нод, неразложимый и простой элементы, фактор по идеалу. На изучение этой штуки в любом учебнике (Зуланке, ван дер Варден) отводится несколько страничек и пять теорем, включая следствия, то есть рассказать можно буквально за несколько минут. Вместе с тем в программе это собираются рассказывать 12 часов.

Ещё восемь часов собираются потратить на симметрические многочлены. Даже у Куроша, написавшего кондовую "Высшую алгебру", на эту тему тратится лишь пара параграфов - не потому что это не важно, а потому что это очень просто. Программа хочет жевать это восемь часов.

На этом вся теория колец заканчивается полностью. Я даже не буду спрашивать, где модули, где нётеровость/артиновость; уже и так понятно, что учебы тут нет. Я просто спрошу: китайская теорема об остатках где?

Во-вторых, сразу после колец шестнадцать часов предлагается выделить на рассказ о ЖНФ. В принципе, тема большая, и такой объём действительно можно заполнить чем-то содержательным, материала хватит. Вот только нет в программе ни нормальной формы Смита, ни абелевых групп и целочисленных решёток, ни связи с рекуррентными уравнениями, ни вообще содержательной теории нормальных форм, ни даже банальной связи с золотым сечением и числами Фибоначчи - есть только "понятие о фробениусовой форме". Таким образом, студенты не узнают, зачем, собственно, нужна эта ЖНФ, и будут считать её только ещё одной забавной разновидностью жонглирования матрицами.

Это был двухсеместровый курс алгебры. Другой алгебры не будет. Таким образом, отучившись на мехмате год, студенты не узнают о:
- бинарных отношениях, алгебраических структурах, фактору по эквивалентности;
- решётках, порядках (частичных, линейных и полных), цепях, индуктивности, о лемме Цорна и других вариантах аксиомы выбора;
- полугруппах и моноидах, кольцах вообще, телах и полях, о классических погружениях моноида в кольцо и кольца в тело;
- нормальных группах, рядах подгрупп, теореме Жордана-Гёльдера, производных рядах и разрешимости;
- действии групп, p-группах и теоремах Силова, о теореме Бёрнсайда и формуле классов, о решении всевозможных задач типа про ожерелья;
- элементарных результатах о конечных группах, о симметрических группах вообще, о знакопеременной группе, о классах сопряженности, о диаграммах Юнга и многочленах Шура, о PGL2 на F5, о внешнем автоморфизме S6;
- кольцах главных идеалов и кольцах Безу; об их связи с евклидовыми кольцами; о факториальных кольцах, о простых и неразложимых элементах, о нод вообще;
- в принципе о делителях нуля, нильпотентах, единицах, простых и максимальных идеалах, об операциях над идеалами, о радикалах и даже о радикале Джекобсона, о факторкольцах и китайской теореме об остатках;
- модулях над полями, телами и кольцами, о свободных модулях, нётеровых и артиновых модулях, о структурной теореме о конечнопорожденных модулях над кольцами главных идеалов и следствиях из неё;
- о тензорных произведениях и типичных операциях с тензорами, инварианте Дэна;
- об алгебрах, внешней алгебре и алгебре Грассмана, о смысле определелителей, об алгебрах многочленов;
- категориях, функторах, естественных преобразованиях, категориях запятой, универсальных свойствах, свободных объектах и представимых функторах, о пределах и копределах, суммах и произведениях, уравнителях и пулбэках;
- ядрах и коядрах, индуктивных и проективных пределах, непрерывных и пропредставимых функторах, решётках подобъектов;
- аддитивных, предабелевых и абелевых категориях, точных функторах, инъективных и проективных объектах, о локализациях;
- теореме Крулля-Ремака-Шмидта о существовании и единственности разложений на неразложимые объекты инъективных объектов в абелевых категориях;
- теореме Крулля-Ремака-Шмидта-Атьи для категорий со стационарными двойными цепями;
- строении инъективных объектов в локально нётеровых категориях и универсальном выводе всех основных структурных теорем (Ласкера-Нётер и о конечнопорожденных над кги, терциарное разложение Лезье-Круазо и т.д.)
- симплициальных комплексах, резольвентах, производных функторах, функторах (ко)гомологии, Ext и Tor;
- представлениях групп и колчанов и их разложениях, характерах, связи с категориями, о лемме Шура и теореме Машке, функторах Кокстера, теореме Габриэля;
- билинейных формах на свободном модуле, теореме Витта и т.д.

Вот вещи, которые просто Must Have для алгебры - то, что я вспомнила навскидку, полей и теории Галуа специально не касалась. Эти вещи можно изучить за два месяца (если с упражнениями). Человек, год отучившийся на мехмате НГУ, не будет знать из этих вещей ни единого термина.
Аноним 21/06/21 Пнд 15:28:58 84687 225
>>84686
Да хуёвая копипаста, выглядит как попытка самоутвердиться. Многое из перечисленного просто пиздёж (что можно проверить, посмотрев на экзаменационные билеты и программу доп семинаров по алгебре на том же сайте), многое просто не нужно вообще либо не осиляемо типичным первокурсником-второкурсником. Читается, как очередной мишин высер (но здесь мы можем быть 100% уверены в том, что это не миша, потому что он с головой ушёл в безмозглую политоту уровня форчана). То, что программа математики не соответствует (как в рассее, так и за океаном) тем знаниям, которые нужно иметь для чтения актуальных статей - это всем известно и устраивать драматический "срыв покровов" - просто нелепо.
Автор поста, очевидно, не имел дела ни с составлением программы для бакалавров, ни с их обучением.
Аноним 21/06/21 Пнд 16:05:41 84688 226
>>84687
>многое просто не нужно вообще
Например?
Аноним 21/06/21 Пнд 16:28:01 84689 227
>>84687
>То, что программа математики не соответствует (как в рассее, так и за океаном) тем знаниям, которые нужно иметь для чтения актуальных статей
Кто тогда эти актуальные статьи пишет, если нет вуза, который научил хотя бы читать их?
Аноним 21/06/21 Пнд 16:33:16 84690 228
Аноним 21/06/21 Пнд 17:43:38 84691 229
2 курс.png 147Кб, 1018x948
1018x948
>>84687
>ни с составлением программы для бакалавров, ни с их обучением
Ну тащемта на матфаке ВШЭ почти так и учат. Программа четырёх первых коллоквиумов: https://docdro.id/8g45Aqa
Листки: https://docdro.id/88MOBDs

Представления традиционно оставляют на второй курс, но иногда рассказывают на первом.

Аноним 21/06/21 Пнд 18:39:12 84692 230
>>84691
Не знаю насчёт "второй курс", но по ссылкам половины нету из топиков что тот анон написал, представления вообще почти не нужны если ты скажем алгебраической геометрией и теорией ходжа занимаешься, не говоря уже о каких-то аналитических вещах.
Аноним 21/06/21 Пнд 18:42:25 84693 231
064d4699dd6754XL.jpg 266Кб, 800x600
800x600
>>84692
Да ничего не нужно, если есть водка.
Аноним 21/06/21 Пнд 18:43:36 84694 232
>>84693
Так представления это и есть водка.
Аноним 21/06/21 Пнд 20:44:51 84699 233
>>84693
Бля картошечка, да еще и с огурчиками. ММмм...
Аноним 28/06/21 Пнд 12:56:34 85051 234
image.png 230Кб, 504x715
504x715
image.png 174Кб, 507x667
507x667
>>80651
О, у Арнольда в книге про это написано.
Аноним 28/06/21 Пнд 16:59:05 85060 235
>>85051
Возможно уже спрашивали но что за книга?
Аноним 28/06/21 Пнд 17:19:44 85061 236
>>85060
Арнольд - Математическое понимание природы.
Аноним 13/09/21 Пнд 09:56:34 87302 237
Да, еще одно полено в костер ВШЭ-мехмат: в ВШЭ учат по листкам. Согласно словам студентов Вышки:

Это самый эффективный педагогический инструмент, когда-либо созданный человеческим умом.

Арнольд учил своих асов-аспов по листкам (конкретное тому подтверждение - Тривиум).

Перельман учился по листкам Рукшина.

Основатель сибирской школы алгебры и логики – Мальцев – учился по листкам.

Шафаревич учился по листкам. В итоге поступил сразу же на последний курс МГУ.

Вся знаменитая группа математиков золотого века Мехмата училась по листкам (см. семинары Лузина).

Сам Лузин учился по листкам (у студента-репетитора).

Софья Ковалевская училась по листкам Вейерштрасса. Впоследствии она активно решала проблемы, поставленные Лондонским матобществом (то тоже были листки).

Чебышёв учился оперировать с многочленами по листкам.

Эндрю Уайлс учился по листкам. В его листке была только одна задача – теорема Ферма (по его словам, он впервые познакомился с теоремой в библиотеке, а затем выписал утверждение к себе в листок).

Гротендик, Лебег, Пикар, Адамар, Вейль – все они учились по листкам.

Все бурбаки коллективно решали листки.

Младшие Бернулли учились по листкам старшего Бернулли. Самый старший Бернулли купил первые математические листки у голландского торговца.

Физики тоже: все ученики Ландау учились по листкам Ландау (см. теорминимум Ландау).

Фейнман учился по листкам в калтехе.

Дирак, Гейзенберг, Борн, Шредингер – все они решали много листков в своих университетах.

У Фарадея был листок, на котором было написано: «объединить электричество с магнетизмом». Он решал много листков из частных и публичных библиотек (он был библиотекарем).

Галилей, Ньютон, Лейбниц – все они учились по листкам, составленными физиками и различного рода арабами, жившими до них.

Аристотель учил Александра Македонского по листкам ВШЭ (!). Из-за этого полководец стал слишком много внимания уделять математике, пренебрегая такими полезными предметами, как география, биология, философия (это было подмечено Арнольдом).

Да и вообще, последними словами Архимеда были: «Не трогай моих листков», что было искаженно переведено современными лингвистами, как «Не трогай моих чертежей».

Евклид учился по листкам, составленным египетскими жрецами.

А вот откуда египетские жрецы нашли первые листки – этого никто не знает. Эту величайшую тайну безуспешно пытаются открыть египтологи. Хотя, после открытий Шампольона, возникло предположение, что египтяне переняли систему листков у древних шумеров.

А вот в мехмате МГУ, как я слышал, по листкам не учат…
Аноним 13/09/21 Пнд 12:04:36 87306 238
НМУ. Секта математиков-маргиналов. Почему секта? Сейчас обосную. Первое. У них есть объект ненависти: они ненавидят классические учебники, а также классическую математическую программу. У них, разумеется, есть "правильные" книги, которыми нужно заменить "неправильные". Они называют одни книги дерьмом и требуют их выкинуть, а другие - священным писанием и единственным авторитетом, основываясь единственно только на популярности книги в НМУ. Второе. Они считают себя заведомо лучше других лишь потому, что принадлежат к касте НМУшников; все, кто не в НМУ - алкоголики, потребители "картофанчика под водовку", все, кто в НМУ, - специалисты. Третье. У них есть легенды о светлом будущем. Разумеется, выпускников НМУ с распростёртыми объятиями встречают в любом из западных университетов (НМУшники доказывают это, приводя в пример двух-трёх человек, в самом деле уехавших из страны). Четвёртое. Какова же реальность? НМУ не выпускает математиков, которые были бы нужны в современной России. НМУ также не выпускает математиков, которые могли бы подтвердить свой диплом на Западе. Выпускник НМУ (а их не так уж и много, два-три человека в год) - это человек, растративший свою жизнь на пустые игры в элитность и не получивший в результате ничего, кроме твёрдой уверенности в собственной б-гоизбранности. Ах да, ещё НМУшник несколько лет учился пить чаёк, решать придуманные другими НМУшниками задачки и рассуждать о том, как плохо жить в России/на этой планете.

Многие НМУшники понимают, что НМУ - это бесполезная секта, и бросают учёбу в этой шараге. Это выдаётся НМУшниками за отбор самых достойных.

НМУшнику, как и всякому агрессивному сектанту (линуксоиду, пгмнутому) сложно противостоять. Он не способен на ведение рациональных дискуссий, поскольку считает любого не-НМУшника унтерменшем и люто-бешено его ненавидит. Если вы не профессиональный психотерапевт - не связывайтесь с НМУшником, высок шанс, что он обратит вас в свою веру.
Аноним 17/09/21 Птн 10:58:35 87387 239
>>84691
Это МАТфак? Почти весь этот кал проходится на 1 курсе вмк, потом программы просто в разные стороны расходятся + все задачи в листках не идут дальше определений. Ебать я проиграл с этого конкурента мехмата, пиздец просто.
Аноним 17/09/21 Птн 13:43:57 87389 240
>>87306
Тухлая паста из обсуждения НМУ на лурочке? Мм, вкусно как. Особенно учитывая, что это всё пиздеж.
Аноним 17/09/21 Птн 15:47:09 87395 241
>>87387
Ну покажи на первом курсе ВМК теорему Машке или лемму Шура.
Аноним 17/09/21 Птн 19:16:24 87399 242
>>87395
Покажи мне конусы на матфаке. Я же говорю, что программы расходятся по темам, собственно теория групп не так уж глубоко рассматривается.
Аноним 17/09/21 Птн 20:29:31 87402 243
>>87395
ну лол, как разница когда проходит теорию представлений - на первом или втором курсе? тем более как уже сказано, она не пригодится половине будущих математиков, см. например дисциплины ICM, по которым читают доклады.
Аноним 17/09/21 Птн 22:59:38 87406 244
группы вмк.png 237Кб, 889x828
889x828
группы вмк2.png 390Кб, 850x890
850x890
матфак2.png 331Кб, 1171x903
1171x903
матфак1.png 254Кб, 947x805
947x805
>>87402
Человек увидел несколько знакомых ему слов. Человек нагло заявил, что пикрелейтед - одинаковы по уровню. Меня раздражает глупость. Не знаю, как тебя.
Аноним 18/09/21 Суб 00:20:15 87407 245
>>87406
не понимаю сравнения на пике. Слева первые занятия по группам, которые можно давать уже в сентябре, справа уже не первые по представлениям групп, наверное из алгебры-2. в питерском мкн такое проходят аж в 6 семестре.
Аноним 18/09/21 Суб 00:26:16 87408 246
>>87407
Слева ВМКшный курс.
>"Избранные вопросы дискретной математики" читается в 5-м семестре (36 ч лекций и 18 ч семинаров). Отчетность - экзамен.
>Курс читает Селезнева Светлана Николаевна
Аноним 18/09/21 Суб 16:51:30 87419 247
>>87406
Что же тебя так рвет, вшивая ты лахта? Скидон отрабатываешь? Я ведь сказал уже, что программы расходятся в разные стороны.
https://dropmefiles.com/007Qn
Аноним 18/09/21 Суб 19:32:47 87420 248
>>87419
Твоё заявление - бред. Доказательства, которыми ты его типа подкрепляешь, - вздор.
>Почти весь этот кал проходится на 1 курсе вмк,
Ничего из матфаковской теории представлений нет на вмк. В файлах по твоей ссылке - простая линейная алгебра (жнф для C и, видимо, попытка её связать с операторными многочленами) и какие-то хаотичные абсолютно элементы общей алгебры, вроде определения характеристики поля. Если ты считаешь, что это то же самое, что алгебра-3, то ты неумён и смыслишь очень мало.
Аноним 18/09/21 Суб 19:33:07 87421 249
>>87419

Интеллигенция из /b подъехала.
Аноним 18/09/21 Суб 19:35:26 87422 250
>>87420
Ты хоть по ссылке-то проходил своей? Я говорю про 1 курс, а не алгебру 3.
Аноним 18/09/21 Суб 19:48:07 87423 251
>>87422
Коллок из поста >>84691, про который ты спорол свою чушь, в основном по алгебре-3. Понятно, что на первом курсе вмк нет ничего из того, что заявлено в этом коллоквиуме.
Аноним 18/09/21 Суб 20:20:26 87424 252
вмк.png 129Кб, 562x810
562x810
>>87423
То есть для более отчётливого понимания. Вот это вот всё с запасом покрывается четырьмя пунктами из третьей главы учебника Городенцева, именно 3.7.4, 3.9.2 (причём только подпунктом 2), 3.13.1 и 3.13.2. А всего в третьей главе - тридцать один пункт. И я уже не говорю, что на ВМК, скорее всего, изучают жнф без всякого упоминания о каких-то там модулях и нормальной форме Смита. Просто делают всё над C, Курош-стайл.

Это как увидеть в одной книге символ Z для целых чисел, в другой книге символ Z для целых чисел и решить поэтому, что книги равноценны. Несмотря на то, что первая книга - учебник алгебры за седьмой класс, а вторая книга - арифметика Серра.
Аноним 18/09/21 Суб 20:32:15 87426 253
>>87424
>скорее всего, изучают жнф без всякого упоминания о каких-то там модулях и нормальной форме Смита. Просто делают всё над C, Курош-стайл.
и правильно делают,
потому что для чего эти модули над абстрактными кольцами вообще нужны? алг. геометрии на вмк, я подозреваю, от природы нет
Аноним 18/09/21 Суб 20:42:36 87428 254
>>87426
Потому что в общем виде оно изучается быстрее. Если уж взялись изучать ЖНФ, то изучать надо по-человечески, а не уродливо. Да и конечнопорождённые абелевы группы на ВМК же классифицируют, в каком-то там спецкурсе. Вроде.
Аноним 18/09/21 Суб 21:46:29 87432 255
>>87423
Я согласен, мы просто не поняли друг друга
Аноним 18/09/21 Суб 23:05:47 87433 256
>>87428
>Потому что в общем виде оно изучается быстрее.
лично у меня это вызывает сомнения

>Если уж взялись изучать ЖНФ, то изучать надо по-человечески, а не уродливо.
и чем же ЖНФ над C такая уродливая по сравнению с какой-то классификацией над какими-то специальными кольцами (главных идеалов)?

>Да и конечнопорождённые абелевы группы на ВМК же классифицируют
очень простой факт, никаких Z-модулей можно не упоминать
Аноним 18/09/21 Суб 23:36:41 87436 257
курош1.png 69Кб, 1650x2479
1650x2479
курош2.png 56Кб, 1650x2479
1650x2479
городенцев1.png 346Кб, 2479x3508
2479x3508
городенцев2.png 288Кб, 2479x3508
2479x3508
>>87433
>такая уродливая
Можно открыть учебники и сравнить. Мне представляется очевидным, что пик1 и пик2 гораздо менее эстетичны, чем пик3 и пик4.

>специальными кольцами (главных идеалов)?
Ну, понятно, что по-хорошему надо было бы доказывать Ласкера-Нётер, но времени у студентов и в самом деле не очень много. А в идеальном мире в курсе просто доказывалась бы Крулля-Ремака-Шмидта в форме Атьи (которая для категорий со стационарными двойными цепями).
Аноним 18/09/21 Суб 23:53:38 87437 258
>>87436
>что пик1 и пик2 гораздо менее эстетичны, чем пик3 и пик4.
только на пик1, пик2 речь про что-то другое
кроме того, они не ссылаются ни на какие теор. 13.1 и "диаграммы Юнга", а используются только прямые вычисления. т.е. сущностей и терминов в них намного меньше.

>Ну, понятно, что по-хорошему надо было бы доказывать Ласкера-Нётер
>А в идеальном мире в курсе просто доказывалась бы Крулля-Ремака-Шмидта в форме Атьи (которая для категорий со стационарными двойными цепями).
не знаю, что в этом хорошего
если хочется завалить неопытного студента горой абстрактного бреда, чтобы он ощутил себя идиотом, когда не проглотит это всё с первого раза, то наверное действительно хорошо
Аноним 19/09/21 Вск 00:10:09 87440 259
>>87437
>пик2 речь про что-то другое
Нет, пик2 - это ключевой элемент доказательства того, что матрицы над полем комплексных подобны тогда и только тогда, когда их характеристические матрицы подобны, - ключевого факта всех этих танцев вокруг ЖНФ. Ну, доказательства по Курошу, конечно.

>"диаграммы Юнга"
Непонятно, почему в кавычках. Какие-то проблемы с диаграммами Юнга?

>горой абстрактного бреда
Ниже этой верхушки шёл слой казённой советской гуманитарии. Это было препротивоестественное (другого, естественного слова тут я не подберу) сообщество, состоявшее из людей простых, тупорылых, всякий «интеллектуализм» искренне ненавидящих, а любящих сало-картошку, водочку и похабные песни. Ну вот взяли уличного, поставили «на гуманитарку». Даже заставили прочесть всякие книжки — как того медведя учили плясать. Он их и прочёл. Даже Маркса-Энгельса, а то и Гегеля, если сильно били. Ненавидел он всю эту тягомотину очень сильно. От души ненавидел.

Натуральный еврейский профессор, сам не чуждый интеллектуальных радостей, мог иной раз и пожалеть умного русского мальчика, «оставить его жить и читать книжки». Сракодрищенский выходец его топил всегда. Именно потому, что искренне ненавидел всё, что этот мальчик любил.
Аноним 19/09/21 Вск 00:23:01 87445 260
>>87440
>матрицы над полем комплексных подобны тогда и только тогда, когда их характеристические матрицы подобны, - ключевого факта всех этих танцев вокруг ЖНФ.
я не помню деталей, но кострикин, например, вроде бы вполне обошёлся без "только тогда". во всяком случае сложных вычислений у него не было. с другой стороны, у него было что-то очень похожее на рисунок, который имеется на твоём пик4, правда, кострикин, конечно, ни про каких юнгов не упоминает

>Ниже этой верхушки
паста не в тему, либо ты не понял мой посыл
Аноним 19/09/21 Вск 02:59:53 87453 261
>>87445
Кострикин (который трёхтомник) - это просто ужас-ужас, а не учебник. Тяжкий, муторный, алогичный - и я это всё могу примерами подкрепить.
>у него было что-то очень похожее на рисунок
Это везде должно быть, где рассказывают про геометрический смысл ЖНФ. Конкретно у Кострикина это появляется в контексте жордановых базисов и циклических клеток. Нарисовано снизу вверх, причём столбики одинаковой высоты Кострикин почему-то не стал рисовать, ограничившись одним столбиком в каждом классе; понятности это не способствует. Более внятное объяснение рисунка есть у Ефимова-Розендорна. Но эта диаграмма и в других контекстах может появляться.

>вполне обошёлся без "только тогда"
Он вообще не формулирует этот критерий и не пользуется лямбда-матрицами. Просто говорит, что над алгебраически замкнутым полем матрица подобна жордановой, и причём единственной. Сначала доказывает, что оператору на пространстве соответствует разложение пространства в прямую сумму корневых, что корневые инвариантны и что операторы, соответствующие корням характеристического многочлена, нильпотентны на их корневых и невырождены на прямой сумме оставшихся. Да, это у него одна большая теорема. Он в неё ещё и анализ размерности вставил. Доказывает тем, что у кольца многочленов есть свойство Безу. Дальше по индукции занудными выкладками доказывает, что у пространства с нильпотентным оператором есть циклический базис. Всё вместе это даёт ему существование ЖНФ. Наконец, единственность ЖНФ он доказывает с помощью системы уравнений на ранги. А затем доказывает существование ещё раз - как раз вот этой вот диаграммой. Все выводы из того факта, что ЖНФ единственна, читатель должен сделать сам, за кадром.

По сути, Кострикин просто два раза повторяет одно и то же доказательство для конечнопорожденных модулей над PID. Мог бы сэкономить себе и читателю время, если бы не побоялся слова "модуль".
Аноним 19/09/21 Вск 05:06:44 87454 262
>>87453
>Кострикин (который трёхтомник) - это просто ужас-ужас, а не учебник.
не знаю, я его почти не читал
правда, признаться, я не очень доверяю твоим оценкам после грёз о "категориях со стационарными двойными цепями"

>Это везде должно быть, где рассказывают про геометрический смысл ЖНФ
само собой, я это и имел в виду: несмотря на все эти модули и кольца главных идеалов, всё равно приходится приводить этот метод, жизнь легче не становится.

>Мог бы сэкономить себе и читателю время, если бы не побоялся слова "модуль".
как будто теорема о классификации это что-то простое
на деле пришлось бы действительно повторить всё то же самое, только не в рамках одного доказательства, а творить предварительно целую теорию.
счастья-то сколько
Аноним 19/09/21 Вск 07:28:56 87455 263
>>87454
>оценкам после грёз о "категориях со стационарными двойными цепями"
Анон хороший, но, видимо, травмированый во время обучения в мгту, оттуда и идеи о радикальных реформах математического образования. Аналогичное случилось и с Димой Павловым, только в его случае это было итмо.
Аноним 19/09/21 Вск 07:49:43 87456 264
теория разложен[...].png 72Кб, 741x683
741x683
>>87454
>категориях со стационарными двойными цепями
Есть условие артиновости/нётеровости - обрыв некоторых цепочек. Тут то же самое.

Двойная цепь в абелевой категории - это набор (An, in, pn), где n пробегает натуральные числа, An - объекты, in - моник из A(n+1) в An, pn - эпик из An в A(n+1). Двойная цепь стационарная, если in и pn изоморфизмы, начиная с некоторого номера N.

Крулль-Ремак-Шмидт-Атья: если в абелевой категории каждая двойная цепь стационарна, то каждый объект обладает разложением на неразложимые прямые слагаемые, и это разложение единственно с точностью до перестановки слагаемых и до их замены на изоморфные.

Из этой теоремы сразу следует много хорошего.

Объект абелевой категории нётеров, если любая его цепочка подобъектов стационарна (т.е. постоянна, начиная с некоторого). Категория локально нётерова, если у неё есть семейство нётеровых образующих. Тип объекта категории - класс всех изоморфных ему объектов. Спектр категории - совокупность типов неразложимых инъективных объектов. Можно думать, что в каждом типе спектра выбран канонический представитель.

В локально нётеровой категории сумма инъективных объектов - инъективный. Каждый ненулевой инъективный - сумма неразложимых инъективных, с точностью до перестановки слагаемых. Таким образом, в локально нётеровой категории между типами инъективных объектов и семействами объектов спектра есть каноническая биекция. Конечные семейства объектов спектра при этом соответствуют инъективным оболочкам нётеровых объектов, и обратно.

Пусть C - локально нётерова категория, A - объект, P(A) - его решётка подобъектов (малая, т.е. множество). Говорят, что на C задана теория разложения, если на объектах категории C задана функция Г такая, что (пикрелейтед).

Теорема. Любую функцию, определённую на спектре C (со значениями в решётке подобъектов A), можно продолжить до теории разложения для C, причём единственным образом.

Дальше с помощью этого факта определяется теория примарного разложения (ака теорема Ласкера-Нётер). В качестве A берём коммутативное нётерово кольцо с единицей, в качестве C - категорию всех A-модулей. Собственно теорема Ласкера-Нётер получается введением нескольких определений (обычный зоопарк модулей) и почти механическим использованием теоремы Кр.Рем.Шм.А. и следствий из неё. И наконец, тривиально получаются основная теорема арифметики, структурная теорема о конечнопорожденных модулях над PID, классификация конечнопорожденных абелевых групп, жорданова форма, и проч., и проч.

Аналогично можно вводить всякие другие теории разложения. Например, вот эта штука http://www.numdam.org/issue/MSM_1963__154__1_0.pdf получается точно так же легко. Достаточно дать определения и сказать несколько слов, связывающих их с Кр.Рем.Шм.А., и уже можно извлекать следствия. Вместо многостраничного мемуара.
Аноним 19/09/21 Вск 07:57:09 87457 265
>>87455
Всё это - древняя гомоалгебра из учебников, которым уже пятьдесят-семьдесят лет. Если хочется именно "реформировать" образование, то нужно учить сразу современным теориям без особой оглядки на прошлое. Но если хочется просто чуть-чуть проапгрейдить преподавание ЖНФ, без революционного перетрахивания всего предмета, то можно вот так.
Аноним 19/09/21 Вск 14:14:11 87470 266
Аноним 19/09/21 Вск 14:36:42 87472 267
>>87456
Топ-даун аппроуч так же убог, как и боттом-ап. Нельзя просто так достать из рукава абстрактных слоников и дедуктивно расставить их на полочке под предлогом, что это "красиво" и "аккуратно". Это приятно читать, это вызывает комфортное ощущение "понимания" - но по факту это такая же хуйня, как и кострикинский зоопарк мутных сущностей.

Вши в силу аутичной традиции дико дрочат на слоников. Комфорт, сухие цветы в вазочке, салфеточка на телевизоре. Теплый чаек, печенье "Юбилейное", ватная баба на чайнике. Золотые шестидесятые, абстрактная чепуха, невинно квотированные евреи. Самиздат, нафталин, старушечьи подштанники.

Тьфу.



Аноним 19/09/21 Вск 14:43:17 87473 268
>>87472
И что же делать?
мимо
Аноним 19/09/21 Вск 18:10:33 87475 269
h-54144.jpg 38Кб, 922x571
922x571
>>87472
>Топ-даун аппроуч так же убог, как и боттом-ап
Аноним 19/09/21 Вск 19:25:59 87478 270
Аноним 19/09/21 Вск 21:21:21 87483 271
>>87473
не выёбываться
Аноним 06/11/21 Суб 11:45:26 89282 272
Теория Манина состоит из трех частей.
Во-первых, он определяет математику как раздел филологии или лингвистики: это наука о формальных преобразованиях одних наборов символов некоторого конечного алфавита в другие при помощи конечного числа специальных "грамматических правил". Отличие математики от живых языков состоит, по Манину, лишь в том, что в ней больше грамматических правил. Например, имеется правило, позволяющее заменять символы "1+2" на "3".
Второй тезис Манина основан на том, что любому человеку с непредвзятым мышлением ясно: подобным переливанием из пустого в порожнее нельзя открыть ничего нового. Если все же в конце и получается что-то интересное, то это означает просто, что оно содержалось уже в исходных данных. Поэтому общество, правительства и т. п. не хотят оплачивать все это бессмысленное переливание из пустого в порожнее. Но математики хотят получать стипендии, гранты и тому подобное. Для этой цели они изобрели университеты и факультеты, где студентов обучают претендовать на открытия (которые им недоступны в силу самого характера их деятельности, как объяснено выше). В этом, по Манину, состоит сущность математического образования: это просто обучение претенциозности.
Третий тезис был добавлен к двум первым после того, как я (во время Международного математического конгресса 1998 г. в Берлине) оспорил первые два. "Некоторые, - пишет Манин, не называя меня, - возражают, утверждая, будто математика полезна в физике, технике и вообще для прогресса человеческой цивилизации. Но они заблуждаются. Чем математика действительно полезна, так это своим огромным вкладом в решение основной проблемы современного постиндустриального человечества. Проблема же эта состоит вовсе не в том, чтобы, как думают некоторые, ускорять прогресс человечества, а напротив, в том, чтобы этот прогресс всемерно тормозить. Математика отвлекает умных людей от действительно опасных для человечества занятий. Если бы вместо проблемы Ферма умники усовершенствовали бы автомобили или самолеты, вреда человечеству было бы больше".
Аноним 06/11/21 Суб 12:17:42 89286 273
>>89282
>Некоторые, - пишет Манин, не называя меня,
как же арнольдом-то резко повеяло, хоть нос затыкай
Аноним 06/11/21 Суб 14:53:13 89296 274
>>89282
>это наука о формальных преобразованиях одних наборов символов некоторого конечного алфавита в другие при помощи конечного числа специальных "грамматических правил"
Лол, у меня такие же идеи были, только я хотел так потроллить когда речь заходит за основания математики. На самом деле существует только один тип данных - это списки. Алфавит задаётся списком, каждая буква однозначно определяется своим положением в списке. Строки - это списки букв конечной длинны. Правила вывода - это правила строковых преобразований, ну и так далее. При этом список как основа всей математики хорошо ещё и тем, что он интуитивно понятен.
Аноним 06/11/21 Суб 16:19:29 89318 275
>>89296
>Лол, у меня такие же идеи были, только я хотел так потроллить
только ты не Ю.И. Манин, и всем наплевать на твои идеи

вообще, мне что-то думается, Арнольд насочинял чего-то от себя как обычно,
вставив в текст побольше местоимений "я" и известных имён выдающихся математиков
Аноним 06/11/21 Суб 17:10:33 89320 276
>>89318
Так ведь и сам Манин ничего нового не сказал, в любом учебнике по логике то же самое написано.
Аноним 06/11/21 Суб 17:13:04 89321 277
>>89320
в данной выдержке манин вообще ничего не сказал,
это всё сказал арнольд, приписав это манину

а учебник по логике и у самого манина есть (я не читал)
Аноним 06/11/21 Суб 18:56:31 89331 278
>>89296
Что такое список?
Аноним 06/11/21 Суб 19:37:26 89335 279
>>89331
..сейчас начнётся
может быть, сразу в основания переместитесь или ещё куда-нибудь?
может, создать тред философия математики,
заодно можно будет обсудить неточность определения предела, 0.(9), определение числа N и т.д.
Аноним 06/11/21 Суб 19:51:37 89337 280
>>89331
Набор, упорядоченный ровно как он записан. Если человек читает этот текст, то он уже знает что такое список. А если он не умеет читать - то и не задаст такой вопрос!
Аноним 11/12/21 Суб 07:38:25 90788 281
Не способен осилить интегралы и диффуры. Прошёл несколько тестов на айсикью наименьшая оценка 126 и то я двое суток не спала, голова не думала, обычно в районе 140. Чо делать?
Аноним 11/12/21 Суб 12:07:26 90792 282
>>90788
А зачем они тебе? Если сдать егэ, достаточно будет натаскаться ну тысяче однотипных примеров
Аноним 17/05/22 Втр 03:16:34 95844 283
FS6qk9rUAAAzV3w.png 282Кб, 527x799
527x799
FS5zRVhX0AAcdsg.png 277Кб, 527x799
527x799
FS5xSoWYAErSqE.jpeg 54Кб, 527x799
527x799
FS6B7iHWQAEHwGh.png 276Кб, 527x799
527x799
Аноним 17/05/22 Втр 03:17:13 95845 284
FS6OuIqWIAQ2tlf.jpeg 114Кб, 1080x1597
1080x1597
FS581HmWIAAVBRF.png 273Кб, 527x799
527x799
FS5qgCgXwAECxK.png 287Кб, 527x799
527x799
When-All-The-Ar[...].png 275Кб, 527x799
527x799
Аноним 15/06/22 Срд 15:42:28 96385 285
>>19672
Это кстати про меня, хоть я ничего сложней квадратного уравнения не решал, но всегда хотел понимать что я делаю, и зачем. И вот я уже закончил универ, работаю программистом, ни одной матрицы даже в универе не посчитал, но в целом лекции слушал и пытался понять что говорят. И вот только спустя годы до меня начинает доходить понимание отрывков того что мне говорили, выстраивается физическая аналогия тех математических действий что мне рассказывали. Действительно, что бы понимать, нужно широкий кругозор иметь, что бы представлять аналогии и с помощью них понимать.
Аноним 16/06/22 Чтв 19:41:58 96414 286
>>96385
>физическая аналогия
ненужна.
Аноним 17/06/22 Птн 11:38:44 96420 287
>>96414
Нужна, ты вектора палочками представляешь, а не векторами
Аноним 18/06/22 Суб 10:57:30 96435 288
>>96420
вывсеврети векторэто упарядоченый набор чисел!!1
Аноним 19/06/22 Вск 11:45:11 96463 289
>>96420

А если это функциональное пространство, какие там палочки?
Аноним 20/06/22 Пнд 12:30:17 96491 290
Аноним 21/06/22 Втр 11:28:26 96507 291
>>96414
Для понимания нужна. Сначала ты складываешь-проецируешь-этц палочки, потом функции.
Аноним 23/06/22 Чтв 00:32:57 96557 292
y = x + 1
x + y = y2 - x2

3 + 4 = 42 - 32
Аноним 08/07/22 Птн 07:05:50 96892 293
Large Cardinals. Lecture notes

http://kamerynjw.net/teaching/2019/math655/part0.pdf
http://kamerynjw.net/teaching/2019/math655/parthalf.pdf
http://kamerynjw.net/teaching/2019/math655/part1.0.pdf
http://kamerynjw.net/teaching/2019/math655/part1.1.pdf
http://kamerynjw.net/teaching/2019/math655/part1.2.pdf
http://kamerynjw.net/teaching/2019/math655/part2.1.pdf
http://kamerynjw.net/teaching/2019/math655/part2.2.pdf
http://kamerynjw.net/teaching/2019/math655/part3.pdf

Part 0 Introduction
Part ½ Logical preliminaries
Part 1.0 Inaccessible cardinals
Part 1.1 Measurable cardinals
Part 1.2 Supercompact cardinals and beyond
Part 2.1 An introduction to forcing
Part 2.2 Actually doing things with forcing
Part 3 Large cardinals and forcing
Аноним 21/07/22 Чтв 12:46:13 97117 294
В качестве курьеза, показывающего, насколько мы были тогда оторваны от всего мира, укажу на сохранившееся в сборнике переводов «Расслоенные пространства» замечание, что изображение спектральной последовательности при помощи прямоугольников из групп, в которых последовательные дифференциалы действуют обобщенным «ходом коня», принадлежит Е. Б. Дынкину (так что соответствующие рисунки в примечаниях к переводу называются диаграммами Дынкина). Когда я оказался в 1965 г. во Франции, то я спросил Серра, знает ли он об этом усовершенствовании теории. Серр долго смеялся — а как же еще можно было проводить вычисления со спектральными последовательностями? Справедливости ради следует отметить, что во французских публикациях (совершенно необходимых читателю) диаграмм не было — возможно, для того, чтобы сделать теорию непознаваемой для непосвященных (но скорее, по обычной беззаботной французской users-unfriendliness).

«Всего полезнее циклы, — говорил он, — но циклы — это нижнее белье, которое нельзя показывать людям: в статьях оставляют одни только классы гомологий»

О ВЛАДИМИРЕ АБРАМОВИЧЕ РОХЛИНЕ
В. И. Арнольд
Аноним 22/07/22 Птн 12:16:58 97153 295
Нашел интересный вопрос, коллеги скорее всего прикалываются. Сколько будет 2/3 от 1/4?
Аноним 27/07/22 Срд 20:08:26 97387 296
Это так. Но есть один нюанс: проблема интеллектуального неравенства людей перпендикулярна проблеме образования. Священным Граалем педагогического мастерства является разработка учебной методики, способной за конечное время вывести любого ученика на пик интеллектуальной продуктивности вне зависимости от уровня его исходных знаний и наличия тех или иных способностей. Учитель обязан исследовать профиль интеллектуальных способностей ученика, оценить сильные и слабые стороны его мышления, выявить все характерные для этого профиля пики и провалы - после чего предоставить своему подопечному набор внутренних или внешних когнитивных инструментов, с помощью которых тот сможет максимально полным образом раскрыть свои изначальные способности и компенсировать свои недостатки. В этом заключается суть профессии педагога.

Физматовский конвейер основан на совершенно иных принципах. Он еще на входе фильтрует людей по IQ, после чего несколько лет фильтрует и ломает их об колено с помощью высосанных из пальца методик преподавания (вроде общепринятой в совке системы Константинова, украденной самим Константиновым с запада) - эффективность которой никогда и никем не была доказана, и которая худо-бедно работает только за счет того, что на конвейер изначально попадает человеческий материал очень высокого качества. Этот материал мог бы достичь тех же самых результатов за счет любого другого метода, либо без всяких методов вообще, чему даже внутри советской конвейерной системы образования найдется множество примеров.

Конвейер не является методикой преподавания. Ты не можешь подать на вход произвольного пятиклассника и получить на выходе достойного бакалавра математики или специалиста хоть в чем бы то ни было. В рамках этой системы нет педагогов. Эта система вообще не умеет учить, в ней нет и никогда не было такого функционала. Она способна только фильтровать, фильтровать, фильтровать... - вываливая в отвалы массы бесполезных магистров, бездарных аспирантов и бесталанных докторов наук, всю жизнь вкалывающих на оборонку и госзаказ или пополняющих бездонные штаты IT-компаний.

И это участь тех, кто все-таки сумел попасть на конвейер. Потому что те, кто не сумел, навсегда исчезают в обычных школах, где их топят, как щенков, в болоте бессмыслицы, в религиозных и секулярных ценностях, в любви к бесконечным деньгам или очередному бессменному вождю, в обыденной жизни, сменке, столовке, физре.

Видишь ли - и безумная популярность очевидных фриков вроде Савватеева, и повсеместное введение цифровизации, и попытки отмены Грефом спецшкол, и восторги школоты по поводу Трушина, и полное поглощение отдельных факультетов IT-корпорациями и даже тот факт, что дети элиты поголовно учатся в частных учебных заведениях на западе - все это части одной системы, элементы грандиозного мега-мемплекса, который полностью накрывает собой медийное пространство вокруг российского образования. Это всего лишь оболочка, мембрана, радужный пузырь, который пытается прикрыть собой пустоту. Шелуха вокруг дырки. Потому что никакого российского образования нет. Ну просто нет.
Аноним 28/07/22 Чтв 14:56:39 97407 297
>>97387
На это уже отвечали в новичковом тренде, зачем эту пургу сюда нести.
Аноним 10/09/22 Суб 20:55:39 98817 298
Миша, привет. Пишет тебе Пьер Делинь из Института перпекстивных исследований в Принстоне. Я много времени не займу. Просто хочу сказать, что ты ебаный мудак, кусающий руку, которая тебе кормит. Ты говно, понял?
Никому не нужна твоя "мудацкая первая культура". Это все пустое. Студентов ВШЭ должны учат дифференциальным уравнениям и механики. Недавно меня приглашали на заседание по реформированию программы ВШЭ, и там все говорили правильно. Диффуры и механика - нужны. Это науки будущего. Подошел к концу век пучков и когомологий.
Ты, говно такое, промыл мозги юным и впечатлительным студентам ВШЭ, и теперь они верят, что "диффуры - говно, пучки - хорошо". Нет, Миша. Говорю тебе, как человек, всю жить занимавшийся алгебраической геометрией. Моим наставником был Александр Гротендик. Все, лавка закрывается, Миша. Гротендик, я, ты, Серр, Бурабки - это прошлый век. Руководство ВШЭ это понимает и учит студентов наукам будущего - дифференциальным уравнениям, механике, теории вероятностей.
Люди понимают, что абстрактная математика не нужна налогоплательщикам, человечеству нужна та математика, которая произведет революцию в прикладных дисциплинах. Нужна математика, которая реформирует мировую финансовую систему, чтобы больше не было нуждающихся, нужна математика, которая даст нам вылететь за пределы галактики, нужна математика, которая поможет нам контролировать погоду.
Твои птенцы тут смеялись, что я, "старый хрыч", всё заседание сидел, "как дурак", и ничего не понимал, просто кивал. А я понимал. Я понимал, что потратил жизнь на никому не нужную абстракщину. И понимал, что не могу допустить, чтобы студенты ВШЭ пошли по моим стопам. Я абсолютно убежден, что ученые умы "Вышки" правы. Что нужны прикладные физические и аналитические науки. Это - будущее математики. Это - будущее человечеста. Это - будущее ВШЭ.

Пошел нахуй, дрочила чертов.

С уважением,

Пьер. Институт перспективных исследований, Принстон, Нью-Джерси.
Аноним 10/09/22 Суб 23:53:30 98821 299
>>70582
Охуеть уже джва года прошло. А я все еще так и не проникся пучками.
Но с другой стороны уже достаточно проникся чтобы было видно что на >>70547 залупа какая то невнятная, больше похожая на определение расслоения

Недавно вот на такую статью наткнулся, может кому будет интересно:
A Very Elementary Introduction to Sheaves
https://arxiv.org/abs/2202.01379

Для любителей CS:
https://reasonablypolymorphic.com/blog/sheafs/index.html
Аноним 11/09/22 Вск 00:41:12 98830 300
>>98821
Раманана прочитал?
Аноним 11/09/22 Вск 00:55:15 98831 301
>>98830
Страничек 10 где-то осилил. Вообще зародилась у меня одна мысль - подавляющее большинство таких "учебников" (не знаю как их назвать, потому что научиться по этой литературе нихуя ничему невозможно на самом деле) имеет смысл читать если ты УЖЕ ЗНАЕШЬ МИНИМУМ ПОЛОВИНУ того что в нем написано. Сам то хоть открывал этого в жопу ебаного Раманана? Еще спиздани что учился по нему чему то.
Аноним 25/03/23 Суб 16:56:15 101989 302
лол типичный матх и наливатели воды уровня аи-чата
>"посоны а вот тут как сказать типа сколько цифр после запятой там я не знаю"
>"работу в десятичной записи можно понимать как морфизмы между коциклами групповых когомологий, да ты собственно не понимаешь про что спрашиваешь, сходи почитай статьи по теме, да и вообще вопрос некорректен, вон ещё серр говорил, что..
Аноним 27/03/23 Пнд 15:04:40 102023 303
>>98817
>нужна математика, которая даст нам вылететь за пределы галактики
Это алгебраическая геометрия, которая работает на теорию связи и помехозащищенного кодирования, см. "алгеброгеометрические коды Гоппы".
Аноним 29/03/23 Срд 11:06:52 102039 304
>Я правильно понимаю что все эти страшно выглядящие диффуры, тройные интегралы и прочее только и нужны чтобы пугать неофитов, а под капотом там всё просто и примитивно?

соответствующая теория не столько примитивная, сколько хорошо разработанная и известная. поэтому осилить её вполне возможно, если приложить определённые усилия

в то же время вычисление конкретных интегралов может быть задачей весьма нетривиальной, не говоря уже о диффурах

вообще диффуры это уже другая история.
за решение уравнений навье-стокса назначена премия в миллион, например
другая задача на миллион, та самая, которую решил гриша перельман, от миллиона отказавшись, тоже содержит в себе нехилые диффуры
в целом, если издалека, то диффуры это одна из самых трудных и малоизвестных математик, в то же время самых важных. так что кто смелый, попробуйте вкатиться в диффуры

те интегралы и диффуры, которые задают на контрольных в вузах, это просто примеры на применение определённых трюков, которые можно даже и не понимать, просто натренировать
Аноним 07/07/23 Птн 05:22:23 103581 305
>Хотелось бы, чтобы уважаемый читатель доказал это в качестве упражнения

Мужик, а шишок под носок тебе не хочется? Хапнуть висюка нет у тебя желания? Ты своего читателя за кого держишь? За шоху, за петыча? Мы тебе чо тут, шестерить должны, утверждения за тебя доказывать? Сидит такой на флексычах: "Читатель то! Читатель сё!" — ПОШЁЛ НА ХУЙ, КЛОУН
Аноним 19/07/23 Срд 22:22:13 104216 306
В начале прошлого века, в постреволюционном Ленинграде, обдолбавшиеся декадентством, обериутской поэзией и русской математической школой шизы решили изобрести новый, достойный советского человека (т. е. гражданина будущей всепланетной Советской Республики) эвристический мемплекс, который можно будет устанавливать в головы бритых наголо коммунистических личинок.

Ну, и изобрели. Создали сеть маткружков, наскребли из дореволюционных компендиумов кучу задач, нагенерировали и обкатали на личинусах относительно свежий материал, раскидали его по топикам (в прогрессорском новоязе это называется "сюжет", в контр-прогрессорском "мотив", смотри не перепутай), сшили в папку и закинули в педагогическое коммьюнити: "работайте, братья".

Разумеется, Коба немедленно перестрелял половину этих фантазирующих о новом человеке матшизов, а вторую половину выморил голодом в блокаду. Но даже кобы не вечны - и когда рябое и усатое чучело издохло, молодой и свеженький Хрущ (с руками по локоть в крови) вдруг понял, что коммунизм к 1980 году, равно как и СССР без бомбуэ невозможен, бомбуэ немыслима без физиков, а физики без математики. О лириках, как сам понимаешь, и речи не было - лирик для комми-прогрессора это заведомо либеральная контра.

Партия дала команду: а ну-ка бля. Недострелянные матшизы тут же взяли под козырек и создали систему добычи теорем из народных масс (она же система математических школ, она же Конвейер, она же Пылесос). Пылесос должен был высасывать таланты из всех уголков нашей обширной родины и направлять работу их интеллекта в сторону создания бомбуэ (собственно, именно по этой причине злые диссидентские языки окрестили Конвейер Канализацией, сливающей таланты в говно). На каждого найденного в ебенях одареныша заводилась папочка в КГБ, и незримые руки гэбни тщательно контролировали его дальнейший жизненный путь: чтобы шарик его судьбы всегда катился в сторону фонящего радиацией "почтового ящика", а не куда-нибудь на Запад.

Но на чем обучать одаренышей, когда Коба полностью наебнул все, что мог? Матшизы приложили палец к необъятному лбу и достали из архива покрытую пылью папку своих ленинградских коллег. "Идея! - воскликнул матшиз Константинов, мир праху его. - А давайте вместо толстых и тяжелых сборников задач использовать маленькие и легкие листочки с задачами - оно же, едрена мать, удобнее в тыщастопицот раз!" И стало так.

Первоначально листочки с задачками действительно были листочками размером в одну страницу - то есть на малом формате нужно было уместить небольшое количество сложных и интересных задач, более-менее равномерно покрывающих собой изученный топик. Иногда подробное изучение топика вообще опускалось и сводилось к паре-тройке предложений, приведенных в начале листочка - а все остальное нужно было доказывать самому. В наши дни листочком называется практически любой набор задач любого содержания на любом количестве страниц. Эвристический мемплекс выродился в методы решения олимпиадных задач - и дегенерирующее коммьюнити олимпиадных композиторов и преподавателей. Конвейер продолжает работу, но уже в пределах только крупных городов - и его основной выхлоп направлен не в сторону ядреных бомб, а в направлении айти-галер или сразу на Запад.

Да и хуй с ним. Комми-прогрессорство благодаря кобам и пыпам давно дискредитировало себя, а для установки эвристического софта гораздо эффективнее работают методы, основанные на когнитивных науках.
Аноним 19/07/23 Срд 22:24:32 104217 307
>Откуда выросла передовая в мире математическая школа, породившая множество выдающихся математиков
Лучшее в мире салфетское образование - это классическая чекистская мулька. Для ее опровержения достаточно сравнить размер выдающегося математического поголовья по обе стороны занавеса или валовой продукт по статьям. Разница между людьми и комми даже не в разы, а на два порядка.

Просто жертвам салфетского конвейера приятно думать о себе, как о наследниках чего-то Виликого и Нибывалого. Признать, что на протяжении трех поколений товарищи майоры тупо сливали их вниз по канализации (сначала в шарашки, потом в академгородки и дубны-бубны, а потом в офисы мейла и яндекса - то есть, прямо скажем, в те же майорские шарашки), им очень больно.

Пилюля слишком горька. Хочется иметь великую и уникальную, сверхпрогрессивную и гиперлиберальную, на две головы обскакавшую и ни на что не похожую великорусскую трансгендерную мультинациональную математическую школу - а по факту плывем все по той же трубе, по тому же говну, в прежнем, указанном еще тов. Берией направлении. Обидно, нэ?
Аноним 26/07/23 Срд 05:08:14 104634 308
Как мартовский кот я приучен к листку, три задачи - три раза в день неукоснительно подхожу я к лотку, ведь пока вербиторебенок во мне еще жив – значит дедлайн по листку не сгорит, а после смерти принимать его будет Бог, и отточенные годами трюки как никто оценит он. На столе три иконки – Вербит, Шень, Гротендик, стопка листков – работа кипит, враги говорят: мог бы в Яндексе штаны просиживать, в ответ молитвою раздается на сердце: дискавери ис э чайлдс привиледж. Скоро осень, пойдет листопад, задачи решатся, пройдется Атья, а пока на плахе треда петух-неосилятор лежит – Константинов Н.Н. с небес довольно урчит.
Аноним 10/08/23 Чтв 17:31:25 105418 309
Как самостоятельно с нуля подготовиться к аспирантуре/докторантуре

Level null:
"Algebra" by I.Gelfand and A.Shen

"How to Prove It: A Structured Approach" D.Velleman

Level one:
"Analysis I" by T.Tao

"Linear Algebra" by K.Hoffman and R.Kunze

"Topics in Algebra" by I.Herstein

"Topology" by J.Munkres

Level two:
"Analysis II" by T.Tao

"Introduction to Measure Theory" by T.Tao

"Complex Analysis" by L.Ahlfors

"Introduction to Smooth Manifolds" by J.Lee

Level three - algebra specialization (MIT OCW courses):
18.700 → 18.701 → 18.702 → 18,704 → 18,715 → 18,721 → 18,782 → 18,705 → 18,706 → 18,725 → 18.726 → 18,745 → 18,755 → 18,783

Level three - analysis and geometry specialization (MIT OCW courses):
18.100 → 18.700 → 18.701 → 18,101 → 18,102 → 18,103 → 18,104 → 18,112 → 18,152 → 18,950 → 18,994 → 18,125 → 18,155 → 18,156 → 18,952 → 18,965 → 18,966

Level three - number theory specialization (MIT OCW courses):
18.700 → 18.701 → 18.702 → 18.781 → 18,704 → 18,721 → 18,782 → 18,784 → 18,705 → 18,725 → 18,783 → 18,785

Level three - probality theory and statistics specialization (MIT OCW courses):
18.065 → 18.100 → 18.600 → 18.700 → 18.701 → 18,102 → 18,103 → 18,112 → 18,200 → 18,211 → 18,615 → 18,642 → 18,650 → 18,125 → 18,675 → 18,676 → 18,677 → 18,338 → 18,416 → 18,424 → 18,655 → 18,657

Level three - topology and geometry specialization (MIT OCW courses):
18.090 → 18.100 → 18.112 → 18.701 → 18.702 → 18.900 → 18.950 → 18,101 → 18,102 → 18,901 → 18,904 → 18,952 → 18,994 → 18,116 → 18,155 → 18,721 → 18,755 → 18,782 → 18,905 → 18,906 → 18,965 → 18,966
Аноним 05/09/23 Втр 21:56:48 107342 310
Когомологии это я не ебу что такое. Могу предположить. "Ко" это так гоорят петухи. "Гомо" значит "гомосексуальный". Значит "логия" от слово "логос", в нашем случае имеется в виду перевод "знание". Значит когомология это учение для петухов, что-то петушиное. Спасибо за внимание
Аноним 06/09/23 Срд 08:38:47 107367 311
>>105418
Опять спискодауны протекли.
Аноним 09/09/23 Суб 06:45:27 107598 312
>>107367
ПРИШЛО ВРЕМЯ СОСТАВИТЬ СПИСОЧЕК БОТА ПО МАТЕМАТИКЕ
СПИСОЧКИ САМИ СЕБЯ НЕ СОСТАВЯТ
СОСТАВЬ ИХ, СОСТАВЬ ИХ ЕЩЁ РАЗ
ЗАЧЕМ МНЕ НУЖНО БОТАТЬ, У МЕНЯ НЕТ ВРЕМЕНИ ЕБАТЬСЯ С ЭТИМ
ЛУЧШЕ ЕЩЁ РАЗ СОСТАВЛЮ СПИСОЧЕК
Аноним 15/09/23 Птн 09:37:38 108204 313
В работе «Ontological revision and quantum mechanics» (2022), говорится о том, что или вы берёте какие-то феномены и начинаете искать математику для выражения этих феноменов, или наоборот — если есть удобная математика, то нужно попросту переопределить онтологию физических объектов, используя эту математику. Скажем, не «частицы» или «волны», а какие-то другие описания материи. В этой работе говорится, что понятие «состояние» понимается как склонность объекта вызывать события у наблюдателей. Дальше работа предлагает онтологически объединить пространства положения и импульса в единое целое, связанное через преобразование Фурье — это же привычно для физики, точно так же ввели пространство-время, и такой ход Эйнштейна не вызвал какого-то особого отторжения у физиков. На основе этой пересмотренной онтологии квантовая механика выводится естественным путем, не опираясь на какие-либо искусственные предположения, такие как «квантовое условие» (по поводу которых до сих пор идут онтологические споры), или математические аппараты, такие как гильбертово пространство305 и самосопряженные операторы. Как следствие, такие острые вопросы, как «проблема измерения», либо разрешаются, либо просто исчезают.
Cамая интересная часть работы «Ontological revision and quantum mechanics» — это попытки рассуждать про онтологию и теорию познания (рациональность и исследования, то есть эпистемологию) в физике, и как находить в одних и тех же математических (если с размерностью — то физических) формулах самое разное понимание. Нет впечатления, что эта работа будет каким-то мейнстримом: на любую новую фундаментальную идею в математике и физике нужно много-много «вязания на спицах»: показывать феномен за феноменом, что каждый из этих феноменов переописывается проще, чем с использованием текущей физики (набор объектов физического мира с их свойствами) и математики (какие математические объекты были выбраны для моделирования физических объектов), или выглядят точнее в новой математике и физике. Без этого никакого широкого распространения теория не получит, но ресурсов на такую обширную работу ни у кого нет.
Тот же David Deutsch не слишком продвигается с constructor theory, ибо недостаточное число людей готовы овладеть новым набором базовых понятий, у Ванчурина и его команды амбиции по нейронной теории вселенной большие — но он и сам признаёт, что вот эта часть проверок и демонстрации того, что описание мира становится проще и объясняет мир точнее, ещё впереди, и непонятно, кто даст на это ресурсы. При таком подходе можно брать за основу описания мира самые разные ментальные объекты. Например, если смотреть на работы Hwe Ik Zhang (автора статьи про ontological revision и quantum mechanics), то можно найти там и работы по математике и физике, стоящими за классическим восточным подходом Инь-Ян И-Цзин, это очень популярная тема на Востоке: если можно класть какую-то онтологию в основании бытия и подтверждать её потом математикой-физикой, типа тех же «струн» или «нейронов», то что б не взять сразу привычный И-Цзин и понятия противоположностей Инь-Ян?! По этой линии предлагается современная переформулировка И-Цзин для биологии. Все эти попытки упираются в то, что какая-то такая «догадка об интересных (то есть с самым разным нетривиальным поведением) ментальных объектах и математике, стоящей за ними (то есть строго описанное, хорошо понятное поведение)» должна привести к смене онтологии/ontology revision — и надо вот просто взять, и всё остальное переписать с опорой на эту догадку, каждый раз показывая, что при переписке не теряется связь с экспериментом, что точность предсказаний мира не ухудшается. Но ресурсов на такое ни у кого нет. Почему у Ньютона и Лейбница получилось, почему у Шрёдингера получилось, почему у Эйнштейна получилось — не очень понятно, это очень интересный вопрос Даже если убрать связь математики и физики, в самой математике тоже радикальные идеи часто не имеют шансов. Например, классический случай Синити Мочизуки, который попытался радикально преодолеть один из барьеров сложности в математике, но не преуспел — слишком мало математиков поняли его идеи. Профессиональному математику понять какую-то новую теорию в математике — это затратить на 1500-2500 страниц 250-500 часов труда, по 5-6 страниц абсолютно нового сложного (с незнакомыми формулами и идеями!) материала в час. Для физиков это будет примерно то же самое, но всё может оказаться сложнее, ибо в физике ещё есть место эксперименту, и с экспериментами тоже надо разобраться, просто «прочесть» не получится. Это означает, что кто-то из авторов теории пишет начальных 1500-2500 страниц изложения картины мира на базе каких-то новых радикальных понятий, а дальше любому новому человеку надо потратить 250-500 часов труда, чтобы продолжить развивать теорию, переписывать на этот новый язык все новые и новые описания, которые и так уже описаны старой теорией. Барьер оказывается ресурсный! Более того, в ходе такого переписывания не факт, что всё окажется лучше: возможно, будут найдены какие-то непреодолимые затруднения, и всё окажется зря, победит в конечном итоге другая идея. А ещё для первых последователей не будет кому проверить их выкладки (в том числе проверить выкладки самого первого автора). Поэтому есть огромные надежды на применение искусственного интеллекта, который сможет работать с такими радикальными идеями: вопервых, порождать такие идеи (типа объединения пространства и времени в пространство-время или объединить положение и импульс в один объект, или представить весь мир как нейроны или струны), а затем пробовать «переписать всю физику, уж сколько можно» на этот новый математический аппарат, подразумевающий выражение новых физических объектов. Но даже для искусственных интеллектов это может быть крайне трудно по ресурсам: представьте себе, что речь идёт о перетолковывании миллионов научных статей!
Но такие радикальные ходы в науке могут быть очень продуктивны, поэтому есть вероятность, что люди будут вкладываться, если демонстрировать универсальность предлагаемого подхода. Так, Ванчурин и его соавторы демонстрируют стандартный ход «аналогии теорий, определяемых на базе структуры математических формул» на смешивании рассуждений для математически выраженных фреймворков термодинамики, теории эволюции и теории нейронных сетей в работе «Thermodynamics of evolution and the origin of life». Это общий ход в рассуждениях такого сорта: «если математика одна и та же, то это проявление общего физического принципа, паттерн поведения вселенной». Так, free energy в информационной теории Шеннона сначала имела смысл не «физической энергии», а совсем другой смысл, просто «в формулах на этом месте в физике стоит обычно энергия, поэтому мы сохранили название в информатике». Но затем жизнь поменялась, и понятие «энергия» в физике стало обозначать ровно вот это — «какой-то физический объект, который ведёт себя так, как ведёт величина из формул, где участвует ментальный/математический объект «энергия»».
Заканчивается вся эта линия рассуждений Ванчунина и соавторов про нейроны и математику обучения как основы физики теорией многоуровневой эволюции как оптимизационного вычисления-познания по типу нейронной сети, «Towards a Theory of Evolution as Multilevel Learning», 2021.
Аноним 17/09/23 Вск 18:05:19 108387 314
>>108204
>Дальше работа предлагает онтологически объединить пространства положения и импульса в единое целое, связанное через преобразование Фурье
это было сделано 60 лет назад в работах В.П. Маслова, например
по-видиму, автор указанных "онтологических ревизий" слова "фазовое пространство" и "симплектическое многообразие" в жизни не слыхал

ему бы посоветоваться со специалистами не помешало прежде чем графоманить
Аноним 07/11/23 Втр 03:33:23 110471 315
5.png 391Кб, 339x638
339x638
7.png 491Кб, 774x586
774x586
Фракталы величайшая ошибка человечества.
В природе не бывает одинаковых деревьев. Каждое несёт отпечаток своей уникальной судьбы. Дерево природное не фрактально в силу его уникальной биографии его симметрия нарушена.

Попытка сказать что природа воспроизводит фракталы есть ложь.
Природа иногда приближается к фракталам.

Попытка навязать это фрактальное упрощение всюду есть ничто иное как попытка вывести из изучения реальную индивидуальность объектов.
Аноним 07/11/23 Втр 13:18:44 110482 316
>>110471

Фрактал не обязательно обладает свойством самоподобия или симметрии. Это что-то нерегулярное, негладкое, рваное и т.д.
Ответить в тред Ответить в тред

Check this out!

Настройки X
Ответить в тред X
15000
Добавить файл/ctrl-v
Стикеры X
Избранное / Топ тредов